Maternal Newborn Final Quizlet

Ace your homework & exams now with Quizwiz!

19. The nurse is discharging five Rh-negative clients from the maternity unit. The nurse knows that the teaching was successful when the client who had which of the following deliveries asks why she must receive a RhoGAM injection? Select all that apply. 1. Abortion at 10 weeks' gestation. 2. Amniocentesis at 16 weeks' gestation. 3. Fetal demise at 24 weeks' gestation. 4. Birth of Rh-negative twins at 35 weeks' gestation. 5. Delivery of a 40-week-gestation, Rh-positive baby.

1 2 3 5 RhoGAM, or Rh immune globulin, is administered to pregnant women at 28 weeks' gestation; after any invasive procedure, like an amniocentesis; after a preterm disruption of a pregnancy, like abortion or placental previa bleed; and after the delivery of an Rh+ infant. Because Rh-negative infants carry no Rh antigen, it is unnecessary to administer RhoGAM to their Rh-negative mothers.

52. A client is to receive a blood transfusion after significant blood loss following a placenta previa delivery. Which of the following actions by the nurse is critical prior to starting the infusion? Select all that apply. 1. Look up the client's blood type in the chart. 2. Check the client's arm bracelet. 3. Check the blood type on the infusion bag. 4. Obtain an infusion bag of dextrose and water. 5. Document the time the infusion begins.

1 2 3 5 The potential for blood transfusion incompatibility is very real. It is essential, therefore, that two health care practitioners check simultaneously to make sure that the client is receiving the correct blood. If any sign of a reaction should develop, the transfusion should be stopped immediately. Only normal saline solution is used as a solution immediately before or after blood administration. Dextrose in water will hemolyze the red blood cells. In addition, a special filtered infusion set must be used.

27. When providing contraceptive counseling to a woman, which of the following factors should the nurse consider? Select all that apply. 1. Age. 2. Obstetric history. 3. Religious beliefs. 4. Employment. 5. Body structure.

1 2 3 Many issues will determine a client's willingness and ability to use family planning methods. For example, if a client is a devout Catholic, she may refuse to use any method other than a natural family planning method. If a client states that she feels she has completed her family, she may be interested in a permanent family planning method. A woman over 35 who smokes should be advised not to use a hormonally based method for health considerations.

110. A nurse has been assigned to circulate during the cesarean section of triplets. Which of the following actions should the nurse take before the birth of the babies? Select all that apply. 1. Count the number of sterile sponges. 2. Document the time of the first incision. 3. Notify the pediatric staff. 4. Perform a sterile scrub. 5. Assemble the sterile instruments.

1 2 3 The circulating nurse is responsible for coordinating the activity in the operating room. He or she is the only member of the team who is able to move freely throughout the room to make telephone calls, obtain needed supplies, maintain the documentation record, and so on. When multiple babies are being birthed, he or she is especially important. The more babies who are birthed at once—e.g., twins, triplets—the more vulnerable the babies are at birth. Multiple-gestation babies are often born preterm and small-for-gestational age. There must be a resuscitation team available for each baby in case emergent care is needed.

73. A couple has delivered a 28-week fetal demise. Which of the following nursing actions are appropriate to take? Select all that apply. 1. Swaddle the baby in a baby blanket. 2. Discuss funeral options for the baby. 3. Encourage the couple to try to get pregnant again in the near future. 4. Ask the couple whether they would like to hold the baby. 5. Advise the couple that the baby's death was probably for the best.

1 2 4 Clients must be encouraged and assisted through the process of grieving and mourning their babies. In addition, as most women will remain on the obstetric unit, there must be a mechanism, like a specific picture placed on the woman's door, for communicating to every department in the hospital, from nursing to housekeeping to dietary, that the client has had a fetal death.

94. The nurse in the obstetrician's office is caring for four 25-week-gestation prenatal clients who are carrying singleton pregnancies. With which of the following clients should the nurse carefully review the signs and symptoms of preterm labor (PTL)? Select all that apply. 1. 38-year-old in an abusive relationship. 2. 34-year-old whose first child was born at 32 weeks' gestation. 3. 30-year-old whose baby has a two-vessel cord. 4. 26-year-old with a history of long menstrual periods. 5. 22-year-old who smokes 2 packs of cigarettes every day.

1 2 5 Even though medical and psychosocial histories are not absolute predictors of preterm labor, there are a number of factors that have been shown to place clients at risk, including pregnancy history of multiple gestations; previous preterm deliveries; cigarette smoking and/or illicit drug use; a number of medical histories like diabetes and hypertension; and social issues like adolescent pregnancy and domestic violence.

55. During a postpartum assessment, it is noted that a G1 P1001 woman, who delivered vaginally over an intact perineum, has a cluster of hemorrhoids. Which of the following would be appropriate for the nurse to include in the woman's health teaching? Select all that apply. 1. The client should use a sitz bath daily as a relief measure. 2. The client should digitally replace external hemorrhoids into her rectum. 3. The client should breastfeed frequently to stimulate oxytocin to reduce the size of the hemorrhoids. 4. The client should be advised that the hemorrhoids will increase in size and quantity with subsequent pregnancies. 5. The client should apply topical anesthetic as a relief measure.

1 2 5 Hemorrhoids are varicose veins of the rectum. They develop as a result of the weight of the gravid uterus on the client's dependent blood vessels. In addition to the actions noted above, the client should be advised to eat high-fiber foods and drink to prevent constipation.

27. A client just spontaneously ruptured membranes. Which of the following factors makes her especially at high risk for having a prolapsed cord? Select all that apply. 1. Breech presentation. 2. Station -3. 3. Oligohydramnios. 4. Dilation 2 cm. 5. Transverse lie.

1 2 5 Once the membranes have ruptured, there are several situations that can increase the possibility of the cord prolapsing, i.e., when the cord slips past the baby and becomes the presenting part. The baby then compresses the cord, preventing the baby from being oxygenated. The situations include malpresentations, like breech and shoulder presentations. A shoulder presentation is the same as a transverse lie. Additional situations that are at high risk for cord prolapse are hydramnios, premature rupture of membranes, and negative fetal station.

87. During a postpartum assessment, the nurse assesses the calves of a client's legs. The nurse is checking for which of the following signs/symptoms? Select all that apply. 1. Pain. 2. Warmth. 3. Discharge. 4. Ecchymosis. 5. Redness

1 2 5 Postpartum clients are high risk for deep vein thrombosis (DVT). At each postpartum assessment the nurse assesses the calves for signs of the complication, i.e., those seen in any inflammatory response: pain, warmth, redness, and edema. If the signs/symptoms are noted, the nurse should request an order from the primary healthcare practitioner for diagnostic tests to be performed, like a Doppler series. Homan's sign is no longer recommended to assess for DVT.

3. A client has just received Hemabate (carboprost) because of uterine atony not controlled by IV oxytocin. For which of the following side effects of the medication will the nurse monitor this patient? Select all that apply. 1. Hyperthermia. 2. Diarrhea. 3. Hypotension. 4. Palpitations. 5. Anasarca.

1 2 Hemabate is an oxytocic agent that acts on the myometrial tissue of the uterus. During the postpartum it acts directly at the site of placental separation to stop uncontrolled bleeding. Hemabate is a type of prostaglandin.

105. A mother tells the nurse that because of family history she is afraid her baby son will develop colic. Which of the following colic management strategies should the parents be taught? Select all that apply. 1. Small, frequent feedings. 2. Prone sleep positioning. 3. Tightly swaddling the baby. 4. Rocking the baby while holding him face down on the forearm. 5. Maintaining a home environment that is cigarette smoke-free.

1 3 4 5 : It is essential to read each possible answer option carefully. Even though it has been shown that colicky babies sometimes find relief when they are placed prone on a hot water bottle, it is not recommended that the babies be left in that position for sleep. It is recommended that healthy babies, whether colicky or not, be placed in the prone position only while awake and while supervised.

42. A client, 32 weeks' gestation with placenta previa, is on total bed rest. The physician expects her to be hospitalized on bed rest until her cesarean section, which is scheduled for 38 weeks' gestation. To prevent complications while in the hospital, the nurse should do which of the following? Select all that apply. 1. Perform passive range-of-motion exercises. 2. Restrict the fluid intake of the client. 3. Decorate the room with pictures of family. 4. Encourage the client to eat a high-fiber diet. 5. Teach the client deep-breathing exercises.

1 3 4 5 Although bed rest is often used as therapy for antenatal clients, it does not come without its complications— constipation, depression, respiratory compromise, muscle atrophy, to name but a few. The nurse must provide preventive care to maintain the health and wellbeing of the client as much as possible.

41. A 12-week-gravid client presents in the emergency department with abdominal cramps and scant dark red bleeding. Which of the following signs/symptoms should the nurse assess this client for? Select all that apply. 1. Tachycardia. 2. Referred shoulder pain. 3. Headache. 4. Fetal heart dysrhythmias. 5. Hypertension.

1 3 4 5 It is essential that the test taker carefully read the weeks of gestation when answering pregnancy-related questions. If the client had been earlier in the first trimester of her pregnancy, the signs and symptoms would also have been consistent with an ectopic pregnancy. It would then have been appropriate to assess for referred shoulder pain as well.

86. A nurse is performing a postpartum assessment on a newly delivered client. Which of the following actions will the nurse perform? Select all that apply. 1. Palpate the breasts. 2. Auscultate the carotid. 3. Check vaginal discharge. 4. Assess the extremities. 5. Inspect the perineum.

1 3 4 5 The best way to remember the items in the postpartum assessment is to remember the acronym BUBBLEHE. The letters stand for: B—breasts; U—uterus; B—bladder; B—bowels and rectum (for hemorrhoids); L—lochia; E—episiotomy; H—Homan's sign; and E—emotional status. But it is important to note that Homan's sign is no longer recommended. Rather, careful inspection of the calves for signs of DVT should be performed.

70. A woman, G1 P0000, is 40 weeks' gestation. Her Bishop score is 4. Which of the following complementary therapies do midwives frequently recommend to clients in similar situations? Select all that apply. 1. Sexual intercourse. 2. Aromatherapy. 3. Breast stimulation. 4. Ingestion of castor oil. 5. Aerobic exercise.

1 3 4 There are many interventions that have been used to increase women's Bishop scores and/or to stimulate labor. Because oxytocin is produced during orgasm and when the breasts are stimulated, intercourse and breast stimulation both can be used as complementary methods of stimulating labor. Castor oil stimulates the bowels. Prostaglandins, which ripen the cervix, are produced as a result of gastrointestinal stimulation. In addition, when ingested, primrose oil converts to prostaglandin in the body. If there is any indication that the baby may be unable to withstand labor, however, these means should not be employed.

84. The nurse is caring for four women who are in labor. The nurse is aware that he or she will likely prepare which of the women for cesarean delivery? Select all that apply. 1. Fetus is in the left sacral posterior position. 2. Placenta is attached to the posterior portion of the uterine wall. 3. Fetus has been diagnosed with meningomyelocele. 4. Client is hepatitis B surface antigen positive. 5. The lecithin/sphingomyelin ratio in the amniotic fluid is 1.5:1.

1 3 Although it is recommended that cesarean section be performed when a mother is affected by two viral illnesses—herpes simplex type 2 (only when active lesions are present) and HIV/AIDS—it is not recommended in the presence of other viral diseases. Hepatitis B is a very serious viral disease, but vertical transmission rates are not significantly different between those babies who are born vaginally and those babies who are born by cesarean section.

111. A client enters the labor and delivery suite. It is essential that the nurse note the woman's status in relation to which of the following infectious diseases? Select all that apply. 1. Hepatitis B. 2. Rubeola. 3. Varicella. 4. Group B streptococcus. 5. HIV/AIDS.

1 4 5 There are several infectious diseases that affect care given during pregnancy, labor and delivery, postpartum, and in the newborn nursery. The hepatitis B status must be assessed to notify the nursery for care needed by the baby. Group B strep status must be assessed to administer needed antibiotics to the mother during labor and to monitor the baby's status in the newborn nursery. The HIV/AIDS status must be assessed to administer needed antiviral medications to the mother in labor and/ or to the baby postdelivery. HIV/AIDS is also an indication for cesarean section delivery.

19. A newborn in the nursery is exhibiting signs of neonatal abstinence syndrome. Which of the following signs/symptoms is the nurse observing? Select all that apply. 1. Hyperphagia. 2. Lethargy. 3. Prolonged periods of sleep. 4. Hyporeflexia. 5. Persistent shrill cry.

1 5 The baby who is exhibiting signs of neonatal abstinence syndrome is craving an addicted drug. The baby's body is agitated because the illicit narcotics he or she has been exposed to are central nervous system depressants and their removal has agitated him or her. The test taker, therefore, should consider symptoms that reflect central nervous system stimulation as correct responses.

82. The nurse is admitting four full-term primigravid clients to the labor and delivery unit. The nurse requests pre-cesarean section orders from the health care practitioner for which of the clients? The client who has: Select all that apply. 1. Cervical cerclage. 2. FH 156 with beat-to-beat variability. 3. Maternal blood pressure of 90/60. 4. Full effacement. 5. Active herpes simplex 2

1 5 The test taker must be able to differentiate in which circumstances a fullterm, otherwise healthy woman, would be unable to deliver vaginally. There are a few absolute indicators for cesarean section: maternal infection with active herpes simplex 2 and HIV/AIDS malpresentation—for example, horizontal lie and breech; previous maternal surgery—e.g., myomectomy; a vertical cesarean scar; some congenital anomalies—e.g., hydrocephalus and meningomyelocele; and other physical conditions, including cervical cerclage in place, obstructive lesions in the lower gynecological system, and complete placenta previa. The test taker should become familiar with each of these.

28. The nurse is caring for a postpartum client who experienced a second-degree perineal laceration at delivery 2 hours ago. Which of the following interventions should the nurse perform at this time? 1. Apply an ice pack to the perineum. 2. Advise the woman to use a sitz bath after every voiding. 3. Advise the woman to sit on a pillow. 4. Teach the woman to insert nothing into her rectum.

1 : A second-degree laceration affects the skin, vaginal mucosa, and underlying muscles. (It does not affect the rectum or rectal sphincter.) Because of the injury, the area often swells, causing pain. Ice packs help to reduce the inflammatory response and numb the area.

96. Four babies with the following conditions are in the well-baby nursery. The baby with which of the conditions is high risk for physiological jaundice? 1. Cephalhematoma. 2. Caput succedaneum. 3. Harlequin coloring. 4. Mongolian spotting.

1 : During the early newborn period, whenever a situation exists that results in the breakdown of red blood cells, the baby is at high risk for hyperbilirubinemia and resulting jaundice. In this case, the baby is at high risk from a cephalhematoma, a collection of blood between the skull and the periosteal membrane. In addition, the neonate is at high risk for hyperbilirubinemia because of the immaturity of the newborn liver.

9. A gravid client with 4+ proteinuria and 4+ reflexes is admitted to the hospital. The nurse must closely monitor the woman for which of the following? 1. Grand mal seizure. 2. High platelet count. 3. Explosive diarrhea. 4. Fractured pelvis.

1 A client who is diagnosed with 4+ proteinuria and 4+reflexes is severely preeclampsia and, therefore, at high risk for becoming eclamptic. Preeclamptic clients are diagnosed with eclampsia once they have had a seizure.

42. Which finding would the nurse view as normal when evaluating the laboratory reports of a 34-week gestation client? 1. Anemia. 2. Thrombocytopenia. 3. Polycythemia. 4. Hyperbilirubinemia

1 By the end of the second trimester, the blood supply of the woman increases by approximately 50%. This increase is necessary for the client to be able to perfuse the placenta. There is a concurrent increase in red blood cell production, but the vast majority of women are unable to produce the red blood cells in sufficient numbers to keep pace with the increase in blood volume. As a result, clients develop what is commonly called "physiological anemia of pregnancy." A hematocrit of 32% is considered normal for a pregnant woman

1. A client has been diagnosed with water intoxication after having received IV oxytocin (Pitocin) for over 24 hours. Which of the following signs/symptoms would the nurse expect to see? 1. Confusion, drowsiness, and vomiting. 2. Hypernatremia and hyperkalemia. 3. Thrombocytopenia and neutropenia. 4. Paresthesias, myalgias, and anemia.

1 Clients who receive oxytocin over a long period of time are at high risk for water intoxication. The oxytocin molecule is similar in structure to the antidiuretic hormone (ADH) molecule. The body retains fluids in response to the medication much the same way it would in response to ADH. The nurse, therefore, should carefully monitor intake and output when clients are induced with oxytocin.

52. A type 1 diabetic gravida has developed polyhydramnios. The client should be taught to report which of the following? 1. Uterine contractions. 2. Reduced urinary output. 3. Marked fatigue. 4. Puerperal rash

1 Clients with hydramnios have excessive quantities of amniotic fluid in their uterine cavities. The excessive quantities likely result from increased fetal urine production, caused by the mother's having periods of hyperglycemia. When the uterus is overextended from the large quantities of fluid, these women are at high risk for preterm labor

5. A 40-week-gestation woman has received Cytotec (misoprostol) for cervical ripening. For which of the following signs/symptoms should the nurse carefully monitor the client? 1. Diarrhea and back pain. 2. Hypothermia and rectal pressure. 3. Urinary retention and rash. 4. Tinnitus and respiratory distress.

1 Cytotec (misoprostol) is a synthetic prostaglandin medication used to ripen the cervix for induction. Gastrointestinal side effects are commonly seen when prostaglandin is used, because the gastrointestinal system is adjacent to the vagina where the medication is inserted. In addition, the nurse must be watchful for signs of labor.

44. On ultrasound, it is noted that the pregnancy of a hospitalized woman who is carrying monochorionic twins is complicated by twin-to-twin transfusion. The nurse should carefully monitor this client for which of the following? 1. Rapid fundal growth. 2. Vaginal bleeding. 3. Projectile vomiting. 4. Congestive heart failure.

1 Fundal growth is accelerated for two reasons: (a) With two babies in utero, uterine growth is increased and (b) the recipient twin—the twin receiving blood from the other twin—often produces large quantities of urine, resulting in polyhydramnios.

1. A 3-day-breastfeeding client who is not immune to rubella is to receive the rubella vaccine at discharge. Which of the following must the nurse include in her discharge teaching regarding the vaccine? 1. The woman should not become pregnant for at least 4 weeks. 2. The woman should pump and dump her breast milk for 1 week. 3. Surgical masks must be worn by the mother when she holds the baby. 4. Antibodies transported through the breast milk will protect the baby

1 If rubella is contracted during pregnancy, the fetus is at very high risk for injury. Whenever gravid clients are found to be nonimmune to rubella— defined as a titer of 1:8 or lower—they are advised to receive the vaccine during the early postpartum period and are counseled regarding the teratogenic properties of the vaccine.

3. A client, 38 weeks' gestation, is being induced with IV oxytocin (Pitocin) for hypertension and oligohydramnios. She is contracting q 3 min × 60 to 90 seconds. She suddenly complains of abdominal pain accompanied by significant fetal heart bradycardia. Which of the following interventions should the nurse perform first? 1. Turn off the oxytocin infusion. 2. Administer oxygen via face mask. 3. Reposition the patient. 4. Call the obstetrician.

1 Oxytocin stimulates the contractility of the uterine muscle. When the muscle is contracted, the blood flow to the placenta is reduced. Whenever there is evidence of fetal compromise and oxytocin is being infused, the intravenous should be stopped immediately to maximize placental perfusion.

59. Which of the following nursing interventions would be appropriate for the nurse to perform to achieve the client care goal: The client will not develop postpartum thrombophlebitis? 1. Encourage early ambulation. 2. Promote oral fluid intake. 3. Massage the legs of the client twice daily. 4. Provide the client with high-fiber foods.

1 Postpartum clients are at high risk for thrombophlebitis because of an increase in the quantity of circulating clotting factors. To prevent clot formation, clients should ambulate as soon as possible after delivery. If they must be bed bound because of complications, the nurse should contact the physician for an order for antiembolic stockings and/or antiembolic pressure boots and have the client perform active range-of-motion exercises.

2. A patient, 32 weeks pregnant with severe headache, is admitted to the hospital with preeclampsia. In addition to obtaining baseline vital signs and placing the client on bed rest, the physician ordered the following four items. Which of the orders should the nurse perform first? 1. Assess deep tendon reflexes. 2. Obtain complete blood count. 3. Assess baseline weight. 4. Obtain routine urinalysis.

1 Preeclampsia is a very serious complication of pregnancy. The nurse must assess for changes in the blood count, for evidence of marked weight gain, and for changes in the urinalysis. By assessing the patellar reflexes first, however, the nurse can make a preliminary assessment of the severity of the preeclampsia. For example, if the reflexes are +2, the client would be much less likely to become eclamptic than a client who has +4 reflexes with clonus.

72. A gravid woman has sickle cell anemia. Which of the following situations could precipitate a vaso-occlusive crisis in this woman? 1. Hypoxia. 2. Alkalosis. 3. Fluid overload. 4. Hyperglycemia.

1 Sickle cell anemia is an autosomal recessive disease. The hemoglobin in the red blood cells of sickle cell clients becomes misshapen when the clients are hypoxic, acidotic, and/or dehydrated. This is a very serious state for the pregnant woman and her fetus. These clients must be cared for immediately with intravenous fluids and methods to reverse the hypoxia and acidosis.

58. A gravid woman, 36 weeks' gestation with type 1 diabetes, has just had a biophysical profile (BPP). Which of the following results should be reported to the obstetrician? 1. One fetal heart acceleration in 20 minutes. 2. Three episodes of fetal rhythmic breathing in 30 minutes. 3. Two episodes of fetal extension and flexion of 1 arm. 4. One amniotic fluid pocket measuring 3 cm.

1 The BPP is a comprehensive assessment geared to evaluate fetal health. In addition to the four items mentioned above, the fetus should exhibit 3 or more discrete body or limb movements in 30 minutes.

2. A 3-day-postpartum client questions why she is to receive the rubella vaccine before leaving the hospital. Which of the following rationales should guide the nurse's response? 1. The client's obstetric status is optimal for receiving the vaccine. 2. The client's immune system is highly responsive during the postpartum period. 3. The client's baby will be high risk for acquiring rubella if the woman does not receive the vaccine. 4. The client's insurance company will pay for the shot if it is given during the immediate postpartum period.

1 The correct answer did not explicitly state that the vaccine is administered during the immediate postpartum period because the woman is not pregnant and is unlikely to become pregnant within the next 4 weeks. But the test taker must know that a woman's obstetric status immediately after delivery is optimal for receiving the medication precisely because she is not pregnant and very unlikely to become pregnant.

89. A 32-week-gestation client states that she "thinks" she is leaking amniotic fluid. Which of the following tests could be performed to determine whether the membranes had ruptured? 1. Fern test. 2. Biophysical profile. 3. Amniocentesis. 4. Kernig assessment.

1 The fern test was so named because when amniotic fluid is viewed under a microscope, it appears as a fern-like image. The image is a reflection of the high estrogen levels in the fluid that create a crystalline pattern. When the fern appears, the nurse can be assured that amniotic fluid is leaking from the amniotic sac.

4. The nurse is evaluating the effectiveness of bed rest for a client with mild preeclampsia. Which of the following signs/symptoms would the nurse determine is a positive finding? 1. Weight loss. 2. 2+ proteinuria. 3. Decrease in plasma protein. 4. 3+ patellar reflexes.

1 The key to answering this question is the test taker's ability to interpret the meaning of mild preeclampsia and to realize that this is an evaluation question. There are two levels of preeclampsia. Mild preeclampsia is characterized by the following signs/symptoms: blood pressure 140/90, urine protein +2, patellar reflexes +3, and weight gain. As can be seen, the values included in answers 2 and 4 are the same as those in the diagnosis. They, therefore, are not signs that the preeclampsia is resolving. Similarly, loss of protein is not a sign of resolution of the disease.

5. A 3-day-postpartum breastfeeding woman is being assessed. Her breasts are firm and warm to the touch. When asked when she last fed the baby her reply is, "I fed the baby last evening. I let the nurses feed him in the nursery last night. I needed to rest." Which of the following actions should the nurse take at this time? 1. Encourage the woman exclusively to breastfeed her baby. 2. Have the woman massage her breasts hourly. 3. Obtain an order to culture her expressed breast milk. 4. Take the temperature and pulse rate of the woman

1 The lactating breast produces milk in response to being stimulated. When a feeding is skipped, milk is still produced for the baby. When the baby is not fed, breast congestion or engorgement results. Not only is engorgement uncomfortable, it also gives the body the message to stop producing milk, resulting in an insufficient milk supply.

53. A pregnant diabetic has been diagnosed with hydramnios. Which of the following would explain this finding? 1. Excessive fetal urination. 2. Recurring hypoglycemic episodes. 3. Fetal sacral agenesis. 4. Placental vascular damage.

1 The majority of amniotic fluid is created as urine by the fetal kidneys. Fetuses of diabetic mothers often experience polyuria as a result of hyperglycemia. If the mother's diabetes is out of control, excess glucose diffuses across the placental membrane, resulting in the fetus becoming hyperglycemic. As a result, the fetus exhibits the classic sign of diabetes—polyuria. If the mother's serum glucose levels are very high during the first trimester, it is likely that the fetus will develop structural congenital defects. Sacral agenesis is one of the most severe of these defects.

58. One nursing diagnosis that a nurse has identified for a postpartum client is: Risk for intrauterine infection r/t vaginal delivery. During the postpartum period, which of the following goals should the nurse include in the care plan in relation to this diagnosis? Select all that apply. 1. The client will drink sufficient quantities of fluid. 2. The client will have a stable white blood cell count. 3. The client will have a normal temperature. 4. The client will have normal-smelling vaginal discharge. 5. The client will take two or three sitz baths each day.

2 3 4 The WBC is elevated during late pregnancy, delivery, and early postpartum, but if it rises very rapidly, the rise is often associated with a bacterial infection. The lochia usually smells "musty." When a client has endometritis, however, the lochia smells "foul." A temperature above 100.4°F after the first 24 hours postpartum is indicative of a puerperal infection.

99. The nurse caring for an infant with a congenital cardiac defect is monitoring the child for which of the following early signs of congestive heart failure? Select all that apply. 1. Palpitations. 2. Tachypnea. 3. Tachycardia. 4. Diaphoresis. 5. Irritability

2 3 4 The term that is most descriptive in the phrase "congestive heart failure" is the word "failure." If the test taker remembers that, because of poor functioning, the heart is failing to oxygenate the body effectively, the test taker can remember the symptoms of the disease. When the body is being starved of oxygen, the body compensates by increasing respirations to take in more oxygen and the pulse rate speeds up to move the oxygenated blood more quickly through the body. Sweating is also a component of the early stages of the disease.

60. A mother is attempting to latch her newborn baby to the breast. Which of the following actions are important for the mother to perform to achieve effective breastfeeding? Select all that apply. 1. Place the baby on his or her back in the mother's lap. 2. Wait until the baby opens his or her mouth wide. 3. Hold the baby at the level of the mother's breasts. 4. Point the baby's nose to the mother's nipple. 5. Wait until the baby's tongue is pointed toward the roof of his or her mouth

2 3 4 The test taker must remember that positioning of a baby at the breast is much different from positioning a bottle-fed baby. For example, even though bottle-fed babies feed effectively while lying on their backs, breastfeeding will be unsuccessful in the same position.

113. The nurse is caring for a 32-week G8 P7007 with placenta previa. Which of the following interventions would the nurse expect to perform? Select all that apply. 1. Daily contraction stress tests. 2. Blood type and cross match. 3. Bed rest with passive range-of-motion exercises. 4. Daily serum electrolyte assessments. 5. Weekly biophysical profiles.

2 3 5 Because clients with placenta previa are at high risk for bleeding from the placental site, it is essential that they be limited in their activity and have blood on hand in case of hemorrhage. In addition, their babies must be monitored carefully for signs of fetal well-being. It would be inappropriate to stimulate contractions because dilation of the cervix would stimulate bleeding.

71. A pregnant woman, 24 weeks' gestation, who has been diagnosed with severe choledocholithiasis is scheduled for a cholecystectomy. In addition to routine surgical and post-surgical care, the nurses should pay special attention to which of the following? Select all that apply. 1. The baby will be delivered by cesarean section at the same time as the cholescystectomy surgery. 2. The woman should be placed in the lateral recumbent position during the surgical procedure. 3. The post-anesthesia care nurse should monitor the woman carefully for nausea and vomiting. 4. The post-anesthesia care nurse should monitor the woman carefully for hemorrhage at the surgical site. 5. Antiembolic stockings should be placed on the woman's legs in the post-anesthesia care unit.

2 3 5 Surgery is performed on a pregnant woman only when absolutely necessary. When it is performed, however, the client's pregnancy hormone levels, the cardiovascular changes of pregnancy, and the size of the gravid uterus all place the client at risk of complications. In addition, of course, the maintenance of the pregnancy itself is at risk because of the surgery.

46. Which of the following pregnant clients is most high risk for preterm premature rupture of the membranes (PPROM)? Select all that apply. 1. 31 weeks' gestation with prolapsed mitral valve (PMV). 2. 32 weeks' gestation with urinary tract infection (UTI). 3. 33 weeks' gestation with twins post-in vitro fertilization (IVF). 4. 34 weeks' gestation with gestational diabetes (GDM). 4. 35 weeks' gestation with deep vein thrombosis (DVT).

2 3 Although the exact mechanism is not well understood, clients who have urinary tract infections are high risk for PPROM. This is particularly important as pregnant clients often have urinary tract infections that present either with no symptoms at all or only with urinary frequency, a complaint of many pregnant clients. Also, clients carrying twins are at high risk for PPROM.

28. A nurse is caring for four prenatal clients in the clinic. Which of the clients is high risk for placenta previa? Select all that apply. 1. Jogger with low body mass index. 2. Primigravida who smokes 1 pack of cigarettes per day. 3. Infertility client who is carrying in vitro triplets. 4. Registered professional nurse who works 12-hour shifts. 5. Police officer on foot patrol.

2 3 The placenta usually implants at a vascular site on the posterior portion of the uterine wall. Two of the women are at high risk for placenta previa. There are 3 placentas nourishing fraternal triplets. Because of the amount of space needed for the placentas, it is not unusual for one to implant near or over the cervical os. The uterine lining of women who smoke is often not well perfused, sometimes resulting in the placenta implanting on or near the cervical os. Women with vascular disease and grand multigravidas are also high risk for placenta previa.

63. Which of the following physical findings would lead the nurse to suspect that a client with severe preeclampsia has developed HELLP syndrome? Select all that apply. 1. +3 pitting edema. 2. Petechiae. 3. Jaundice. 4. +4 deep tendon reflexes. 5. Elevated specific gravity.

2 3 The test taker must be able to discriminate between symptoms of severe preeclampsia and HELLP syndrome. If the nurse remembers what each of the letters in HELLP stands for, he or she can determine which of the responses is correct.

50. The nurse has taught a new admission to the postpartum unit about pericare. Which of the following indicates that the client understands the procedure? Select all that apply. 1. The woman performs the procedure twice a day. 2. The woman washes her hands before and after the procedure. 3. The woman sits in warm tap water for ten minutes three times a day. 4. The woman sprays her perineum from front to back. 5. The woman mixes warm tap water with hydrogen peroxide

2 4 : A postpartum client is taught to spray warm tap water with nothing added on the perineum, from front to back, after each toileting and whenever she changes her peripads. She should also be taught to wash her hands before and after the procedure.

74. An obese gravid woman is being seen in the prenatal clinic. The nurse will monitor this client carefully throughout her pregnancy because she is high risk for which of the following complications of pregnancy? Select all that apply. 1. Placenta previa. 2. Gestational diabetes. 3. Abruptio placentae. 4. Preeclampsia. 5. Chromosomal defects.

2 4 Because clients who enter pregnancy obese are at such high risk for gestational diabetes, many obstetricians skip the glucose challenge test and automatically schedule a glucose tolerance test at approximately 24 weeks' gestation. As a result, the complication is discovered much earlier and intervention can begin much earlier. The patients are also carefully monitored for signs and symptoms of preeclampsia.

76. A baby is suspected of having esophageal atresia. The nurse would expect to see which of the following signs/symptoms? Select all that apply. 1. Frequent vomiting. 2. Excessive mucus. 3. Ruddy complexion. 4. Abdominal distention. 5. Pigeon chest.

2 4 With esophageal atresia, the esophagus ends in a blind pouch. In addition, there is usually a fistula connecting the stomach to the trachea. These babies are at high risk for respiratory compromise because they can aspirate the large quantity of oral mucus. The neonatologist should be notified whenever esophageal atresia is suspected.

2. The nurse is assessing the laboratory report of a 40-week gestation client. Which of the following values would the nurse expect to find elevated above prepregnancy levels? Select all that apply. 1. Glucose. 2. Fibrinogen. 3. Hematocrit. 4. Bilirubin. 5. White blood cells.

2 5 During the latter part of the third trimester, coagulation factors increase in preparation for delivery. It is the body's means of protecting itself against a large loss of blood at delivery. In addition, the white blood cell count rises as a means of protecting the body from infection

53. A nurse is caring for the following four laboring patients. Which client should the nurse be prepared to monitor closely for signs of postpartum hemorrhage (PPH)? Select all that apply. 1. G1 P0000, delivered a fetal demise at 29 weeks' gestation. 2. G2 P1001, prolonged first stage of labor. 3. G2 P0010, delivered by cesarean section for failure to progress. 4. G3 P0200, delivered vaginally a 42-week, 2,200-gram neonate. 5. G4 P3003, with a succenturiate placenta.

2 5 The muscles of the uterus of a client who has experienced a prolonged first stage of labor are fatigued. In the postpartum period, therefore, they may fail to contract fully enough to control bleeding at the site of placental separation. A succenturiate placenta is characterized by one primary placenta that is attached via blood vessels to satellite lobe(s). These clients must be monitored carefully for postpartum hemorrhage.

20. A woman, 39 weeks' gestation, is admitted to the delivery unit with vaginal warts from human papillomavirus. Which of the following actions by the nurse is appropriate? 1. Notify the health care practitioner for a surgical delivery. 2. Follow standard infectious disease precautions. 3. Notify the nursery of the imminent delivery of an infected neonate. 4. Wear a mask whenever the perineum is exposed.

2 Although HPV is a sexually transmitted infection and it can in rare instances be contracted by the neonate from the mother, the Centers for Disease Control and Prevention do not recommend that cesarean section be performed merely to prevent vertical transmission of HPV

39. A 36-week gestation gravid client is complaining of dyspnea when lying flat. Which of the following is the likely clinical reason for this complaint? 1. Maternal hypertension. 2. Fundal height. 3. Hydramnios. 4. Congestive heart failure

2 As the uterus enlarges, the woman's organs are affected. At 36 weeks, the fundus is at the level of the xiphoid process. The diaphragm is elevated and the lungs are displaced. When a client lies flat she has difficulty breathing. Most women use multiple pillows at night for sleep. Whenever caring for a pregnant woman, the nurse should elevate the head of the bed.

117. An ultrasound has identified that a client's pregnancy is complicated by hydramnios. The nurse would expect that an ultrasound may show that the baby has which of the following structural defects? 1. Pulmonic stenosis. 2. Tracheoesophageal fistula. 3. Ventriculoseptal defect. 4. Developmental hip dysplasia.

2 Babies swallow the amniotic fluid while in utero. When there is a surplus of fluid, ultrasounds may be performed to check for defects in the fetal gastrointestinal system.

62. A gestational diabetic, who requires insulin therapy to control her blood glucose levels, telephones the triage nurse complaining of dizziness and headache. Which of the following actions should the nurse take at this time? 1. Have the client proceed to the office to see her physician. 2. Advise the client to drink a glass of juice and then call back. 3. Instruct the client to inject herself with regular insulin. 4. Tell the client immediately to telephone her medical doctor

2 Because the signs and symptoms of hyperglycemia and hypoglycemia are very similar, it is important for the nurse to err on the side of caution. If the client should be hypoglycemic, this is a medical emergency. Drinking a glass of orange juice will stabilize the glucose in the woman's body. If she is hyperglycemic, the juice may increase the glucose levels, but not significantly. A blood glucose assessment can be done and insulin can be administered, if needed, shortly after consuming the juice.

94. Prenatal teaching for a pregnant woman should include instructions to do which of the following? 1. Refrain from touching her pet bird. 2. Wear gloves when gardening. 3. Cook pork until medium well done. 4. Avoid sleeping with the dog.

2 Clients should be advised to wear gloves when gardening because cat feces can carry the toxoplasmosis protozoa. Feral and outdoor domestic cats are nondiscriminating about where they urinate and defecate. They easily could be using the vegetable garden for a cat box. As such, it is also very important for everyone, and especially pregnant women, to wash fresh fruits and vegetables before eating them.

115. A client has been admitted with a diagnosis of hyperemesis gravidarum. Which of the following orders written by the primary health care provider is highest priority for the nurse to complete? 1. Obtain complete blood count. 2. Start intravenous with multivitamins. 3. Check admission weight. 4. Obtain urine for urinalysis.

2 Clients who are vomiting repeatedly are energy depleted, vitamin depleted, electrolyte depleted, and often dehydrated. It is essential that the client receive her IV therapy as quickly as possible. The other orders should be completed soon after the IV is started.

100. A 4-day-old baby born via cesarean section is slightly jaundiced. The laboratory reports a bilirubin assessment of 6.0 mg/dL. Which of the following would the nurse expect the neonatologist to order for the baby at this time? 1. To be placed under phototherapy. 2. To be discharged home with the parents. 3. To be prepared for a replacement transfusion. 4. To be fed glucose water between routine feeds.

2 Hemolytic jaundice is seen within the first 24 hours of life. A neonatologist would be concerned about the health of the baby with a bilirubin of 6 mg/dL during that time frame. Physiological jaundice, on the other hand, is seen in about 50% of healthy full-term babies, with bilirubin levels rising after the first 24 hours and peaking at 3 to 5 days. A level of 6 mg/dL at 4 days, therefore, is well within normal limits.

105. A woman with a history of congestive heart disease is 36 weeks pregnant. Which of the following findings should the nurse report to the primary health care practitioner? 1. Presence of striae gravidarum. 2. Dyspnea on exertion. 3. 4-pound weight gain in a month. 4. Patellar reflexes +2.

2 It is important for the test taker to know that pregnancy is a significant stressor on the cardiac system. Women who enter the pregnancy with a history of cardiac problems must be monitored very carefully not only by the obstetric practitioner but also by an internist or cardiologist. The nurse must be vigilant in observing for signs of cardiac failure, including respiratory and systemic congestion.

12. The physician has ordered Prepidil (dinoprostone) for four gravidas at term. The nurse should question the order for which of the women? 1. Primigravida with Bishop score of 4. 2. Multigravida with late decelerations. 3. G1 P0000 contracting every 20 minutes × 30 seconds. 4. G6 P3202 with blood pressure 140/90 and pulse 92.

2 It is important to remember that although the fetus of a pregnant woman may be at term, it is not always safe for labor contractions to be stimulated. Although Prepidil is not directly used for induction, it is an agent that promotes cervical ripening in preparation for labor. A baby who is exhibiting signs of poor uteroplacental blood flow is likely to be compromised further by the addition of the medication.

33. A woman with a diagnosis of ectopic pregnancy is to receive medical intervention rather than a surgical interruption. Which of the following intramuscular medications would the nurse expect to administer? 1. Decadron (dexamethasone). 2. Amethopterin (methotrexate). 3. Pergonal (metotropin). 4. Prometrium (progesterone).

2 Methotrexate is an antineoplastic agent. Even if the test taker were unfamiliar with its use in ectopic pregnancy but was aware of the action of methotrexate, he or she could deduce its efficacy here. Methotrexate is a folic acid antagonist that interferes with DNA synthesis and cell multiplication. The conceptus is a ball of rapidly multiplying cells. Methotrexate interferes with that multiplication, killing the conceptus and, therefore, precluding the need for the client to undergo surgery.

38. The nurse is assessing the midline episiotomy on a postpartum client. Which of the following findings should the nurse expect to see? 1. Moderate serosanguinous drainage. 2. Well-approximated edges. 3. Ecchymotic area distal to the episiotomy. 4. An area of redness adjacent to the incision

2 The best tool to use when assessing any incision is the REEDA scale. The nurse assesses for: R—redness, E—edema, E—ecchymosis, D—drainage, and A—poor approximation. If there is evidence of any of the findings, they should be documented and monitored and reported.

92. A nurse is about to administer the ophthalmic preparation to a newly born neonate. Which of the following is the correct statement regarding the medication? 1. It is administered to prevent the development of neonatal cataracts. 2. The medicine should be placed in the lower conjunctiva from the inner to outer canthus. 3. The medicine must be administered immediately upon delivery of the baby. 4. It is administered to neonates whose mothers test positive for gonorrhea during pregnancy.

2 The eye prophylaxis clouds the vision of the neonate. Even though it is state law in all 50 states that the medication be given, it is best to delay the instillation of the medication for an hour or so after birth so that eye contact and parent-infant bonding can occur during the immediate postuterine period.

4. An induction of a 42-week gravida with IV oxytocin (Pitocin) is begun at 0900 at a rate of 0.5 milliunits per minute. The woman's primary physician orders: Increase the oxytocin drip by 0.5 milliunits per minute every 10 minutes until contractions are every 3 minutes × 60 seconds. The nurse refuses to comply with the order. Which of the following is the rationale for the nurse's action? 1. Fetal distress has been noted in labors when oxytocin dosages greater than 2 milliunits per minute are administered. 2. The relatively long half-life of oxytocin can result in unsafe intravascular concentrations of the drug. 3. It is unsafe practice to administer oxytocin intravenously to a woman who is carrying a postdates fetus. 4. A contraction duration of 60 seconds can lead to fetal compromise in a baby that is postmature.

2 The half-life (the time it takes half of a medication to be metabolized by the body) of oxytocin is relatively long— about 15 minutes. And at least 3 half-lives usually elapse before therapeutic responses are noted. Increasing the infusion rate too rapidly, therefore, can lead to hyperstimulation of the uterine muscle and consequent fetal distress.

15. A client is admitted in labor with spontaneous rupture of membranes 24 hours earlier. The fluid is clear and the fetal heart rate is 124 with moderate variability. Which assessment is most important for the nurse to make at this time? 1. Contraction frequency and duration. 2. Maternal temperature. 3. Cervical dilation and effacement. 4. Maternal pulse rate.

2 The test taker must remember that the uterine cavity is a sterile space and the vaginal vault is an unsterile space. When membranes have ruptured over 24 hours, there is potential for pathogens to ascend into the uterine cavity and infection to result. Elevated temperature is a sign of infection.

64. A nurse is counseling a woman about postpartum blues. Which of the following should be included in the discussion? 1. The father may become sad and weepy. 2. Postpartum blues last about a week or two. 3. Medications are available to relieve the symptoms. 4. Very few women experience postpartum blues

2 The test taker must not confuse the three psychological changes that mothers may experience postpartum: postpartum blues, postpartum depression, and postpartum psychosis. Postpartum blues is a normal phenomenon related to fatigue, hormonal shifts, and the enormous responsibility of becoming a mother. Postpartum depression and postpartum psychosis are pathological conditions that only some women experience

63. A 6-month-old child is being seen in the pediatrician's office. The child was born preterm and remained in the neonatal intensive care unit for the first 5 months of life. The child is being monitored for 5 chronic problems. Which of the following problems are directly related to the prematurity? Select all that apply. 1. Bronchopulmonary dysplasia. 2. Cerebral palsy. 3. Retinopathy. 4. Hypothyroidism. 5. Seizure disorders.

3 4 Even if the test taker were unfamiliar with the expected lab findings of a neonate that had been born after living with an aging placenta, deductive reasoning could assist the test taker to choose the correct response. Aging placentas function poorly, and therefore the fetuses receive less nutrition and oxygenation. The baby's body, therefore, must compensate for the losses by metabolizing glycogen and lipid stores and by producing increased numbers of red blood cells. The neonate, therefore, is often polycythemic, hypoglycemic, and hypolipidemic

21. A woman has just been admitted to the emergency department subsequent to a head-on automobile accident. Her body appears to be uninjured. The nurse carefully monitors the woman for which of the following complications of pregnancy? Select all that apply. 1. Placenta previa. 2. Transverse fetal lie. 3. Placental abruption. 4. Severe preeclampsia. 5. Preterm labor

3 5 The fetus is well protected within the uterine body. The musculature of the uterus and the amniotic fluid provide the baby with enough cushioning to withstand minor bumps and falls. A major automobile accident, however, can cause anything from preterm premature rupture of the membranes, to preterm labor, to a ruptured uterus, to placental abruption. The nurse should especially monitor the fetal heartbeat for any variations.

102. A nurse hears a heart murmur on a full-term neonate in the well-baby nursery. The baby's color is pink while at rest and while feeding. Which of the following cardiac defects is consistent with the nurse's findings? Select all that apply. 1. Transposition of the great vessels. 2. Tetralogy of Fallot. 3. Ventricular septal defect. 4. Pulmonic stenosis. 5. Patent ductus arteriosus.

3 5 The names of cardiac defects are very descriptive. Once the test taker remembers the pathophysiology of each of the defects, it becomes clear how the blood flow is affected. Of the choices in this question, the defects that are acyanotic defects, i.e., defects that allow blood to enter the lungs to be oxygenated, are the VSD and the PDA.

45. A nurse is performing an assessment on four 22-week-pregnant clients. The nurse reports to the obstetrician that which of the clients may be carrying twins? 1. The client who states that she feels huge. 2. The client with a weight gain of 13 pounds. 3. The client whose fundal height measurement is 26 cm. 4. The client whose alpha-fetoprotein level is one-half normal.

3 After 20 weeks' gestation, the nurse would expect the fundal height to be equal to the number of weeks of the woman's gestation. Because the fundal height is 4 cm above the expected 22 cm, it is likely that the woman is either having twins or has polyhydramnios

94. A neonate is to receive the hepatitis B vaccine in the neonatal nursery. Which of the following must the nurse have available before administering the injection? 1. Hepatitis B immune globulin in a second syringe. 2. Sterile water to dilute the vaccine before injecting. 3. Epinephrine in case of severe allergic reactions. 4. Oral syringe because the vaccine is given by mouth.

3 Although vaccinations are administered relatively routinely, they are not without their potential side effects. One very serious side effect is anaphylaxis. Therefore, the nurse should always have epinephrine available in case of a severe reaction

30. An obstetrician is performing an amniotomy on a gravid woman in transition. Which of the following assessments must the nurse make immediately following the procedure? 1. Maternal blood pressure. 2. Maternal pulse. 3. Fetal heart rate. 4. Fetal fibronectin level.

3 Amniotomy, as the word implies, is the artificial rupture of the amniotic sac. During the procedure, there is a risk that the umbilical cord may become compressed. Because there is no direct way to assess cord compression, the nurse must assess the fetal heart rate for any adverse changes

3. A nurse is counseling a preeclamptic client about her diet. Which should the nurse encourage the woman to do? 1. Restrict sodium intake. 2. Increase intake of fluids. 3. Eat a well-balanced diet. 4. Avoid simple sugars.

3 Clients with preeclampsia are losing albumin through their urine. They should eat a well-balanced diet with sufficient protein to replace the lost protein. Even though preeclamptic clients are hypertensive, it is not recommended that they restrict salt—they should have a normal salt intake—because during pregnancy the kidney is salt sparing. When salt is restricted, the kidneys become stressed.

96. A client, 37 weeks' gestation, has been advised that she is positive for group B streptococci. Which of the following comments by the nurse is appropriate at this time? 1. "The doctor will prescribe intravenous antibiotics for you. A visiting nurse will administer them to you in your home." 2. "You are very high risk for an intrauterine infection. It is important for you to check your temperature every day." 3. "The bacteria are living in your vagina. They will not hurt you but we will give you medicine in labor to protect your baby from getting sick." 4. "This bacteria causes scarlet fever. If you notice that your tongue becomes very red and that you feel feverish you should call the doctor immediately."

3 Group B strep can cause serious neonatal disease. Babies are at high risk for meningitis, sepsis, pneumonia, and even death. IV antibiotics are administered to the laboring mother every 4 hours to decrease the colonization in the mother's vagina and rectum. In addition, the antibiotics cross the placenta and act as a prophylaxis for the baby.

29. A woman has been diagnosed with a ruptured ectopic pregnancy. Which of the following signs/symptoms is characteristic of this diagnosis? 1. Dark brown rectal bleeding. 2. Severe nausea and vomiting. 3. Sharp unilateral pain. 4. Marked hyperthermia.

3 The most common location for an ectopic pregnancy to implant is in a fallopian tube. Because the tubes are nonelastic, when the pregnancy becomes too big, the tube ruptures. Unilateral pain can develop because only one tube is being affected by the condition, but some women complain of generalized abdominal pain.

25. Which of the following findings should the nurse expect when assessing a client, 8 weeks' gestation, with gestational trophoblastic disease (hydatidiform mole)? 1. Protracted pain. 2. Variable fetal heart decelerations. 3. Dark brown vaginal bleeding. 4. Suicidal ideations.

3 The most important thing to remember when answering questions about hydatidiform mole is the fact that, even though a positive pregnancy test has been reported, there is no "pregnancy." The normal conceptus develops into two portions—a blastocyst, which includes the fetus and amnion, and a trophoblast, which includes the fetal portion of the placenta and the chorion. In gestational trophoblastic disease (hydatidiform mole), only the trophoblastic layer develops; no fetus develops. With the proliferation of the chorionic layer, the client is high risk for gynecological cancer.

14. A client with mild preeclampsia who has been advised to be on bed rest at home asks why doing so is necessary. Which of the following is the best response for the nurse to give the client? 1. "Bed rest will help you to conserve energy for your labor." 2. "Bed rest will help to relieve your nausea and anorexia." 3. "Reclining will increase the amount of oxygen that your baby gets." 4. "The position change will prevent the placenta from separating."

3 This question requires the nurse to have a clear understanding of the pathology of preeclampsia. Only with an understanding of the underlying disease can the test taker be able to remember the rationale for many aspects of client care. The vital organs of preeclamptic clients are being poorly perfused as a result of the abnormally high blood pressure. When a woman lies on her side, blood return to the heart is improved and the cardiac output is also improved. With improved cardiac output, perfusion to the placenta and other organs is improved.

10. A client is admitted to the hospital with severe preeclampsia. The nurse is assessing for clonus. Which of the following actions should the nurse perform? 1. Strike the woman's patellar tendon. 2. Palpate the woman's ankle. 3. Dorsiflex the woman's foot. 4. Position the woman's feet flat on the floor

3 When clients have severe preeclampsia, they are often hyperreflexic and develop clonus. To assess for clonus, the nurse should dorsiflex the foot and then let the foot go. The nurse should observe for and count any pulsations of the foot. The number of pulsations is documented. The higher the number of pulsations there are, the more irritable the woman's central nervous system is

29. A client is complaining of severe back labor. Which of the following nursing interventions would be most effective? 1. Assist mother with childbirth breathing. 2. Encourage mother to have an epidural. 3. Provide direct sacral pressure. 4. Move the woman to a hydrotherapy tub.

3 Whenever a laboring woman complains of severe back labor, it is very likely that the baby is lying in the occiput posterior position. Every time the woman has a contraction, the head is pushed into the coccyx. When direct pressure is applied to the sacral area, the nurse is providing counteraction to the pressure being exerted by the fetal head.

26. A female client asks the nurse about treatment for human papilloma viral warts. The nurse's response should be based on which of the following? 1. An antiviral injection cures approximately fifty percent of cases. 2. Aggressive treatment is required to cure warts. 3. Warts often spread when an attempt is made to remove them surgically. 4. Warts often recur a few months after a client is treated.

4 : Genital warts are caused by the human papillomavirus. There are more than 100 viral types of HPV. Most of them are harmless, but unfortunately, some high-risk types can cause cancer. Some of the topical treatments for genital warts can be applied at home by the individual or can be administered by a practitioner. Surgery and cryotherapy, also used to treat warts, must be performed by a skilled practitioner

29. A woman had a cesarean section yesterday. She states that she needs to cough but that she is afraid to. Which of the following is the nurse's best response? 1. "I know that it hurts but it is very important for you to cough." 2. "Let me check your lung fields to see if coughing is really necessary." 3. "If you take a few deep breaths in, that should be as good as coughing." 4. "If you support your incision with a pillow, coughing should hurt less."

4 Clients with abdominal incisions experience significant postoperative pain. And because their abdominal muscles have been incised, the pain is increased when the clients breathe in and cough. Bracing the abdominal muscles with a pillow or a blanket helps to reduce the discomfort.

65. An insulin-dependent diabetic woman will require higher doses of insulin as which of the following pregnancy hormones increases in her body? 1. Estrogen. 2. Progesterone. 3. Human chorionic gonadotropin. 4. Human placental lactogen.

4 During the first trimester, the insulin needs of a woman with type 1 diabetes are usually low. Once the diabetic client enters the second trimester, however, insulin demands increase. One of the most important reasons that insulin demands increase is the increasingly higher levels of human placental lactogen that are found in the mother's bloodstream.

8. A client has severe preeclampsia. The nurse would expect the primary health care practitioner to order tests to assess the fetus for which of the following? 1. Severe anemia. 2. Hypoprothrombinemia. 3. Craniosynostosis. 4. Intrauterine growth restriction.

4 Perfusion to the placenta drops when clients are preeclamptic because the client's hypertension impairs adequate blood flow. When the placenta is poorly perfused, the baby is poorly nourished. Without the nourishment provided by the mother through the umbilical vein, the fetus's growth is affected.

6. When during the latent phase of labor should the nurse assess the fetal heart pattern of a low-risk woman, G1 P0000? Select all that apply. 1. After vaginal exams. 2. Before administration of analgesics. 3. Periodically at the end of a contraction. 4. Every ten minutes. 5. Before ambulating.

1 2 3 5 Except for invasive procedures, assessment of the fetal heart pattern is the only way to evaluate the well-being of a fetus during labor. The fetal heart pattern should, therefore, be assessed whenever there is a potential for injury to the baby or to the umbilical cord. At each of the times noted in the scenario—vaginal exam, analgesic administration, contraction, and ambulation—either the cord could be compressed or the baby could be compromised.

1. The nurse is discussing the neonatal blood screening test with a new mother. The nurse knows that the teaching was successful when the mother states that the test screens for the presence in the newborn of which of the following diseases? Select all that apply. 1. Hypothyroidism. 2. Sickle cell disease. 3. Galactosemia. 4. Cerebral palsy. 5. Cystic fibrosis.

1 2 3 5 It is important to realize that neonatal screening is state-specific. Each state determines which diseases will be screened for. The March of Dimes and other groups have recommended that at least 29 inborn diseases be screened for in all states

10. A nurse is doing a newborn assessment on a new admission to the nursery. Which of the following actions should the nurse make when evaluating the baby for developmental dysplasia of the hip (DDH)? Select all that apply. 1. Grasp the baby's legs with the thumbs on the inner thighs and forefingers on the outer thighs. 2. Gently adduct and abduct the baby's thighs. 3. Palpate the trochanter during hip rotation. 4. Place the baby in a fetal position. 5. Compare the lengths of the baby's legs.

1 2 3 5 The test taker should review assessment skills. To assess for developmental dysplasia of the hip, the Ortolani and the Barlow tests are performed. The order of the steps of the Ortolani procedure is (a) the nurse places the baby on its back; (b) the nurse grasps the baby's thighs with a thumb on the inner aspect and forefingers over the trochanter; (c) with the knees flexed at 90° angles, the hips are abducted; and (d) the nurse palpates the trochanter to assess for hip laxity. The Barlow test is performed by: (a) adducting the baby's legs; (b) gently pushing the legs posteriorly; and (c) feeling to note any slippage of the trochanter out of the acetabulum. Galeazzi sign can also be performed.

45. A nurse is working in the prenatal clinic. Which of the following findings seen in third-trimester pregnant women would the nurse consider to be within normal limits? Select all that apply. 1. Leg cramps. 2. Varicose veins. 3. Hemorrhoids. 4. Fainting spells. 5. Lordosis.

1 2 3 5 There are a number of physical complaints that are "normal" during pregnancy. There are interventions, however, that can be taught to help to alleviate some of the discomforts. The test taker should be familiar with patient education information that should be conveyed regarding the physical complaints of pregnancy. For example, clients who complain of hemorrhoids should be encouraged to eat high-fiber foods and drink fluids to produce softer stools. The softer stools should decrease the irritation of the hemorrhoids.

19. A female African American baby has been admitted into the nursery. Which of the following physiological findings would the nurse assess as normal? Select all that apply. 1. Purple-colored patches on the buttocks and torso. 2. Bilateral whitish discharge from the breasts. 3. Bloody discharge from the vagina. 4. Sharply demarcated dark red area on the face. 5. Deep hair-covered dimple at the base of the spine

1 2 3 : A multiple-response type of question is often a more difficult type of question to answer than is a standard multiple-choice item because there is not simply one correct response to the question. The test taker must look at each answer option to see whether or not it accurately answers the stem of the question. In this question, purple-colored patches, a whitish discharge from the breasts, and a bloody discharge in a female African American neonate are all considered normal.

45. A nurse is providing anticipatory guidance to a couple regarding the baby's immunization schedule. Which of the following statements by the parents shows that the teaching by the nurse was successful? Select all that apply. 1. The first hepatitis B injection is given by 1 month of age. 2. The first polio injection will be given at 2 months of age. 3. The MMR (measles, mumps, and rubella) immunization should be administered before the first birthday. 4. Three DTaP (diphtheria, tetanus, and acellular pertussis) shots will be given during the first year of life. 5. The Varivax (varicella) immunization will be administered after the baby turns one year of age.

1 2 4 5 Many recommendations are time specific. The CDC changes immunization recommendations when new research emerges. The test taker should periodically review reliable sites like www.CDC.gov (Centers for Disease Control and Prevention) and www.aap. org (American Academy of Pediatrics) to check recommendations.

83. Which of the following behaviors would indicate to a nurse that a gravid woman may be being abused? Select all that apply. 1. Denies that any injuries occurred, even when bruising is visible. 2. Gives an implausible explanation for any injuries. 3. Gives the nurse eye contact while answering questions. 4. Allows her partner to answer the nurse's questions. 5. Frequently calls to change appointment times.

1 2 4 5 Nurses must use all their senses when interviewing clients. A physical assessment should be conducted as well as questions asked to check for evidence of abuse. In addition, the client's communications must be critically assessed. Women who always defer to their partners may be exhibiting a sign of abuse. Plus, the history provided by the client and/or her partner must be evaluated for its credibility. If any injuries do not coincide with the story provided, the nurse must investigate the situation further

9. A nurse is caring for a laboring woman who is in transition. Which of the following signs/symptoms would indicate that the woman is progressing into the second stage of labor? Select all that apply. 1. Bulging perineum. 2. Increased bloody show. 3. Spontaneous rupture of the membranes. 4. Uncontrollable urge to push. 5. Inability to breathe through contractions.

1 2 4 : It is important that the test taker clearly understands the difference between the three phases of the first stage of labor and the three stages of labor. The three phases of the first stage of labor—latent, active, and transition— are related to changes in cervical dilation and maternal behaviors. The three stages of labor are defined by specific labor progressions—cervical change to full dilation (stage 1), full dilation to birth of the baby (stage 2), birth of the baby to birth of the placenta (stage 3)

55. A bottle-feeding mother is providing a return demonstration of how to burp the baby. Which of the following would indicate that the teaching was successful? Select all that apply. 1. The woman gently strokes and pats her baby's back. 2. The woman positions the baby in a sitting position on her lap. 3. The woman waits to burp the baby until the baby's feeding is complete. 4. The woman states that a small amount of regurgitated formula is acceptable. 5. The woman remarks that the baby does not need to burp after trying for one full minute.

1 2 4 : It is important to distinguish between babies who are bottle-fed and those who are breastfed. Breastfed babies usually ingest much less air than do bottle-fed babies. Breastfed babies should be burped at least once in the middle of their feeds, whereas bottle-fed babies should be burped every 1 /2 to 1 ounce.

84. A client is being seen following a sexual assault. A rape examination is being conducted. Which of the following specimens may be collected from the victim during the examination? Select all that apply. 1. Buccal swab for genetic analysis. 2. Samples of pubic hair. 3. Toenail scrapings. 4. Samples of head hair. 5. Sputum for microbiological analysis

1 2 4 In many ways, a rape examination is another form of invasion for a woman who has been raped. The examiner, only after being given permission by the victim, will take a number of samples, including those mentioned above. Other samples that may be obtained include vaginal smears, any and all clothing worn by the victim, and pictures of any and all injuries. If the perpetrator were to go to trial, it would be far in the future. By that time the victim will no longer have any outward signs of assault. It is important to provide the prosecution with as much evidence as possible if a conviction is to be obtained. It is essential to remember, however, that the victim must be allowed to refuse any examinations

84. Which of the following nonpharmacological interventions recommended by nurse midwives may help a client at full term to go into labor? Select all that apply. 1. Engage in sexual intercourse. 2. Ingest evening primrose oil. 3. Perform yoga exercises. 4. Eat raw spinach. 5. Massage the breast and nipples.

1 2 5 : If the test taker were unfamiliar with nonpharmacological induction methods, he or she could make some educated guesses by remembering that pharmacological medications for labor induction are prostaglandins and oxytocin. When a woman has an orgasm during intercourse, she releases oxytocin. Nipple and breast massage also stimulate oxytocin production. And evening primrose oil contains a fatty acid that converts into a prostaglandin compound.

76. Which of the following actions would the nurse expect to perform immediately before a woman is to have regional anesthesia? Select all that apply. 1. Assess fetal heart rate. 2. Infuse 1,000 mL of Ringer's lactate. 3. Place the woman in the Trendelenburg position. 4. Monitor blood pressure every 5 minutes for 15 minutes. 5. Have the woman empty her bladder.

1 2 5 Before any medication, whether analgesia or anesthesia, is administered during labor, the fetal heart should be assessed to make sure that the baby is not already compromised. Before regional anesthesia administration, a liter of fluid should be infused to increase the woman's vascular fluid volume. This will help to maintain her blood pressure after the epidural insertion. And the woman's bladder should be emptied because she will not have the sensation of a full bladder once the epidural is in place.

46. A nurse is advising the parents of a newborn regarding when they should call their pediatrician. Which of the following responses show that the teaching was effective? Select all that apply. 1. If the baby repeatedly refuses to feed. 2. If the baby's breathing is irregular. 3. If the baby has no tears when he cries. 4. If the baby is repeatedly difficult to awaken. 5. If the baby's temperature is above 100.4°F.

1 4 5 The test taker must judge each answer option independently of the others when completing a multipleresponse item. These items require more comprehensive knowledge because there is not simply one best response, but rather many correct answers.

62. The parents and their full-term, breastfed neonate were discharged from the hospital. Which behavior 2 days later indicates a positive response by the parents to the nurse's discharge teaching? Select all that apply. 1. The parents count their baby's diapers. 2. The parents measure the baby's intake. 3. The parents give one bottle of formula every day. 4. The parents take the baby to see the pediatrician. 5. The parents time the baby's feedings.

1 4 : In 2004, the AAP published a statement recommending that babies be seen by the pediatrician at 3 to 5 days of age to assess for the presence of jaundice, dehydration, or other complications. Because most babies are discharged on day 2 of life, they need to be taken to the pediatrician within 3 days of discharge

2. A woman, whose menstrual cycle is 35 days long, states that she often has a slight pain on one side of her lower abdomen on day 21 of her cycle. She wonders whether she has ovarian cancer. Which of the following is the nurse's best response? 1. "Women often feel a slight twinge when ovulation occurs." 2. "You should seek medical attention as soon as possible since ovarian cancer is definitely a possibility." 3. "Ovarian cancer is unlikely because the pain is not a constant pain." 4. "It is more likely that such pain indicates an ovarian cyst because pain is more common with that problem."

1 : Although the stem includes the fact that the woman is concerned about ovarian cancer, this question is actually testing what the test taker knows about ovulation. The test taker should try not to be confused by extraneous information included in a stem.

28. While evaluating the fetal heart monitor tracing on a client in labor, the nurse notes that there are fetal heart decelerations present. Which of the following assessments must the nurse make at this time? 1. The relationship between the decelerations and the labor contractions. 2. The maternal blood pressure. 3. The gestational age of the fetus. 4. The placement of the fetal heart electrode in relation to the fetal position.

1 : Decelerations are defined by their relationship to the contraction pattern. It is essential that the nurse determine which of the three types of decelerations is present. Early decelerations mirror contractions, late decelerations develop at the peak of contractions and return to baseline well after contractions are over, and variable decelerations can occur at anytime and are unrelated to contractions

66. A woman is taking Fosamax (alendronate) for osteoporosis. The nurse should advise the woman about which of the following when taking the medication? 1. Remain upright for 30 minutes after taking the medication. 2. Take only after eating a full meal. 3. Take medication in divided doses 3 times each day. 4. Do not break or crush the tablet.

1 : Fosamax must be consumed with a full glass of water on an empty stomach. It is especially important that the client sit upright for at least 30 minutes after taking the medication because severe upper gastrointestinal irritation can result when reclining. Esophageal irritation, ulceration, and erosions can develop when the medication is taken improperly.

27. A triage nurse answers a telephone call from the male partner of a woman who was recently diagnosed with cervical cancer. The man is requesting to be tested for human papillomavirus (HPV). The nurse's response should be based on which of the following? 1. There is currently no approved test to detect HPV in men. 2. A viral culture of the penis and rectum is used to detect HPV in men. 3. A Pap smear of the meatus of the penis is used to detect HPV in men. 4. There is no need for a test because men do not become infected with HPV

1 : Some gay men do have anal Pap smears done to attempt to detect cancer cells in the rectum. This practice is controversial and has not been accepted

22. An infertile woman has been diagnosed with endometriosis. She asks the nurse why that diagnosis has made her infertile. Which of the following explanations is appropriate for the nurse to make? 1. "Scarring surrounds the ends of your tubes, preventing your eggs from being fertilized by your partner's sperm." 2. "You are producing insufficient quantities of follicle-stimulating hormone that is needed to mature an egg every month." 3. "Inside your uterus is a benign tumor that makes it impossible for the fertilized egg to implant." 4. "You have a chronic infection of the vaginal tract that makes the secretions hostile to your partner's sperm."

1 : This question is essentially a knowledge-level question. All of the answer options relate to infertility problems, but only one is specifically related to endometriosis. It is important to have an understanding of gynecological issues since many do affect a woman's fertility.

15. When assessing the psychological adjustment of an 8-week gravida, which of the following would the nurse expect to see signs of? 1. Ambivalence. 2. Depression. 3. Anxiety. 4. Ecstasy

1 Even women who stop taking birth control pills to become pregnant are often startled and ambivalent when they actually get pregnant. This is not pathological. The women usually slowly accept the pregnancy and, by 20 weeks' gestation, are happy and enthusiastic about the prospect of becoming a mother.

57. The nurse is developing a standard care plan for the administration of Mifeprex (misepristone/misoprostol; formerly known as RU-486). Which of the following information should the nurse include in the plan? 1. Women should be evaluated by their health care practitioners 2 weeks after taking the medicine. 2. This is the preferred method for terminating an ectopic pregnancy when an intrauterine device is in place. 3. The only symptom clients should experience is bleeding 2 to 3 days after taking the medicine. 4. Women who experience no bleeding within 3 days should immediately take a home pregnancy test.

1 Mifeprex is available for use for terminating unwanted pregnancies, for completing incomplete spontaneous abortions, and for terminating ectopic pregnancies. If the medicine should be ineffective and the pregnancy survives, there is a strong possibility that the fetus will be damaged. It is very important, therefore, that the client be assessed to make sure that she truly aborted the conceptus

5. Which statement by the client indicates that she understands the teaching provided about the intrauterine device (IUD)? 1. "The IUD can remain in place for a year or more." 2. "I will not menstruate while the IUD is in." 3. "Pain during intercourse is a common side effect." 4. "The device will reduce my chances of getting infected."

1 Not only does the IUD not reduce the likelihood of a woman developing a pelvic infection, there are clients who are particularly at high risk for pelvic inflammatory disease following insertion of an IUD. Women who have multiple sex partners or who have a recent history of a sexually transmitted infection should be considered at high risk for infection. It is recommended that the IUD be placed in these women with caution.

27. While caring for a client in the transition phase of labor, the nurse notes that the fetal monitor tracing shows average short-term and long-term variability with a baseline of 142 beats per minute (bpm). What should the nurse do? 1. Provide caring labor support. 2. Administer oxygen via face mask. 3. Change the client's position. 4. Speed up the client's intravenous

1 The baseline fetal heart variability is the most important fetal heart assessment that the nurse makes. If the baby's heart rate shows average variability, the nurse can assume that the baby is not hypoxic or acidotic. In addition, the normal heart rate of 142 is reassuring.

32. A client asks the nurse about the gamete intrafallopian transfer (GIFT) procedure. Which of the following responses would be appropriate for the nurse to make? 1. Fertilization takes place in the woman's body. 2. Zygotes are placed in the fallopian tubes. 3. Donor sperm are placed in a medium with donor eggs. 4. A surrogate carries the infertile woman's fetus.

1 The best way for the test taker to remember the various forms of infertility therapies is to remember the definitions of the components. For example, when GIFT is being discussed, the term "gamete" (G) refers to the male or female reproductive cell—that is, the sperm or ovum. When ZIFT is being considered, the term "zygote" (Z) refers to the fertilized ovum. The prefix "intra" means "within" and the term "fallopian" refers to the fallopian tube. When GIFT (or ZIFT) is discussed, the method of transfer into the fallopian tube is via laparoscope.

71. The nurse is counseling a woman who has been diagnosed with mild osteoporosis. Which of the following lifestyle changes should the nurse recommend? Select all that apply. 1. Eat yellow and orange vegetables. 2. Go on daily walks. 3. Stop smoking. 4. Consume dairy products. 5. Sleep at least eight hours a night.

2 3 4 There are a number of factors that clients are unable to control in relation to the development of osteoporosis—for example, sex (women are more at risk than are men), age (older women are more at risk than younger women), and genetics (family history plays a role). Any client who is at risk because of the preceding factors should be especially counseled to eat well, stop smoking, drink in moderation, and get daily exercise.

5. A mother asks the nurse to tell her about the responsiveness of neonates at birth. Which of the following answers is appropriate? Select all that apply. 1. "Babies have a poorly developed sense of smell until they are 2 months old." 2. "Babies respond to all forms of taste well, but they prefer to eat sweet things like breast milk." 3. "Babies are especially sensitive to being touched and cuddled." 4. "Babies are nearsighted with blurry vision until they are about 3 months of age." 5. "Babies respond to many sounds, especially to the high-pitched tone of the female voice."

2 3 5 : Many parents and students believe that babies are incapable of receptive communication. On the contrary, they are amazingly able. The test taker must review the abilities of neonates to respond appropriately to questions and to teach parents about the abilities of their newborns.

13. A client is in the 10th week of her pregnancy. Which of the following symptoms would the nurse expect the client to exhibit? Select all that apply. 1. Backache. 2. Urinary frequency. 3. Dyspnea on exertion. 4. Fatigue. 5. Diarrhea.

2 4 During the first trimester, the body undergoes a number of important changes. The embryo is developing, the hormones of the body are increasing, and the maternal blood supply is increasing. To accomplish each of the tasks, the body uses energy. The mother is fatigued not only because the body is undergoing great change but also because the thyroid gland has not caught up with the increasing energy demands. In addition, because the organs are confined within the bony pelvis, the enlarging uterus prevents the bladder from expanding with large quantities of urine. As a result, the woman needs to urinate much more frequently than she did prior to becoming pregnant.

24. The nursery charge nurse is assessing a 1-day-old female on morning rounds. Which of the following findings should be reported to the neonatologist as soon as possible? Select all that apply. 1. Blood in the diaper. 2. Grunting during expiration. 3. Deep red coloring on one side of the body with pale pink on the other side. 4. Lacy and mottled appearance over the entire chest and abdomen. 5. Flaring of the nares during inspiration.

2 5 Pseudomenses is seen in many 1-day-old female neonates. Although mottling and the harlequin sign can be present in emergent situations, they are usually normal findings. Expiratory grunting and nasal flaring, however, are not normal. Respiratory difficulties always need to be assessed fully.

41. A client, in her third trimester, is concerned that she will not know the difference between labor contractions and normal aches and pains of pregnancy. How should the nurse respond? 1. "Don't worry. You'll know the difference when the contractions start." 2. "The contractions may feel just like a backache, but they will come and go." 3. "Contractions are a lot worse than your pregnancy aches and pains." 4. "I understand. You don't want to come to the hospital before you are in labor."

2 : Labor contractions often begin in a woman's back, feeling much like a backache. The difference is that labor contractions are intermittent and rhythmic. The client should be advised to attend to any pains that come and go and time them. She may be beginning the labor process.

34. A woman in her third trimester advises the nurse that she wishes to breastfeed her baby, "but I don't think my nipples are right." Upon examination, the nurse notes that the client has inverted nipples. Which of the following actions should the nurse take at this time? 1. Advise the client that it is unlikely that she will be able to breastfeed. 2. Refer the client to a lactation consultant for advice. 3. Call the labor room and notify them that a client with inverted nipples will be admitted. 4. Teach the woman exercises to evert her nipples

2 : Research on eversion exercises has shown that they are not effective, plus breast manipulation can bring on contractions since oxytocin production is stimulated. Lactation consultants are breastfeeding specialists. A lactation consultant would probably recommend that the client wear breast shields in her bra. The shields are made of hard plastic and have a small hole through which the nipple everts.

22. A woman has been diagnosed with primary syphilis. Which of the following physical findings would the nurse expect to see? 1. Cluster of vesicles. 2. Pain-free lesion. 3. Macular rash. 4. Foul-smelling discharge

2 : Syphilis is caused by a spirochete and, like other spirochetal illnesses, has a 3-stage course. The first stage of the disease is the chancre stage. A chancre is a small, round, painless lesion that will disappear, even without treatment, after a month or so. If the client is not treated, the disease will progress to stage 2, during which a reddish brown rash, usually on the palms and soles; sores on the mucous membranes; and flu-like symptoms develop. If the client is still left untreated, the disease will progress to stage 3, the symptoms of which often appear years later: dementia, paralysis, numbness, and blindness. The damage caused in the tertiary stage of syphilis is not reversible

95. A woman is carrying dizygotic twins. She asks the nurse about the babies. Which of the following explanations is accurate? 1. During a period of rapid growth, the fertilized egg divided completely. 2. When the woman ovulated, she expelled two mature ova. 3. The babies share one placenta and a common chorion. 4. The babies will definitely be the same sex and have the same blood type.

2 : The best way for the test taker to differentiate between monozygotic twinning and dizygotic twinning is to remember the meaning of the prefixes to the two words. "Mono" means "1." Monozygotic twins, therefore, originate from one fertilized ovum. The babies have the same DNA; therefore, they are the same sex. They share a placenta and chorion. "Di" means "2." Dizygotic twins arise from 2 separately fertilized eggs. Their genetic relationship is the same as if they were siblings born from different pregnancies.

8. The nurse plans to provide anticipatory guidance to a 10-week gravid client who is being seen in the prenatal clinic. Which of the following information should be a priority for the nurse to provide? 1. Pain management during labor. 2. Methods to relieve backaches. 3. Breastfeeding positions. 4. Characteristics of the newborn

2 : This 10-week gravid client will be entering the second trimester in a couple of weeks. As the uterine body grows, the client is likely to experience backaches. It is appropriate for the nurse to provide information about this possi - bility and ways to relieve them.

63. Which of the following clients, who are all seeking a family planning method, is the best candidate for birth control pills? 1. 19-year-old with multiple sex partners. 2. 27-year-old who bottle feeds her newborn. 3. 29-year-old with chronic hypertension. 4. 37-year-old who smokes one pack per day

2 Birth control pills that contain both estrogen and progesterone are inappropriate for clients who breastfeed because the estrogen inhibits milk production. There is no such contraindication for mothers who bottle feed. It is important to remember, however, that women who breastfeed can use progestin-only pills.

85. The nurse is providing acupressure to provide pain relief to a woman in labor. Where is the best location for the acupressure to be applied? Select all that apply. 1. On the malleolus of the wrist. 2. Above the patella of the knee. 3. On the medial aspect of the lower leg. 4. At the top one third of the sole of the foot. 5. Below the medial epicondyle of the elbow

3 4 Complementary therapies have been shown to be of value in a number of clinical situations, including labor. The specific acupressure point on the leg is located about 3 cm above the inner malleolus in the calf region. Strong pressure placed at these points has been shown to reduce the pain of labor contractions.

37. A neonate, who is being admitted into the well-baby nursery, is exhibiting each of the following assessment findings. Which of the findings should the nurse report to the primary health care provider? Select all that apply. 1. Harlequin sign. 2. Extension of the toes when the lateral aspect of the sole is stroked. 3. Elbow moves past the midline when the scarf sign is assessed. 4. Slightly curved pinnae of the ears that are slow to recoil. 5. Telangiectatic nevi.

3 4 The test taker should not be confused by the mixing of technical terms and descriptions of findings. Even though technical terms were included, the correct responses are actually descriptions—in this case, a description of the scarf sign and the immature pinnae of the ears as seen in preterm babies.

86. The pediatrician writes the following order for a term newborn: Vitamin K 1 mg IM. Which of the following responses provides a rationale for this order? 1. During the neonatal period, babies absorb fat-soluble vitamins poorly. 2. Breast milk and formula contain insufficient quantities of vitamin K. 3. The neonatal gut is sterile. 4. Vitamin K prevents hemolytic jaundice

3 : It is important for the test taker to review how vitamin K is synthesized by the intestinal flora. Because the neonate is deficient in intestinal flora until 1 week of age, he or she is unable to manufacture vitamin K until that time. Vitamin K is important, especially for babies who will be circumcised, because it is needed to activate coagulation factors synthesized in the liver

25. An asymptomatic woman is being treated for HIV infection at the women's health clinic. Which of the following comments by the woman shows that she understands her care? 1. "If I get pregnant, my baby will be HIV positive." 2. "I should have my viral load and antibody levels checked every day." 3. "Since my partner and I are both HIV positive, we use a condom." 4. "To be safe, my partner and I engage only in oral sex."

3 : The human immunodeficiency virus is prone to mutation. It is important that clients use condoms whenever they have intercourse because if the virus mutates and the client becomes infected with two strains of virus, the progression to AIDS is hastened.

29. In which of the following situations would it be appropriate for the father to place the baby in the en face position to promote neonatal bonding? 1. The baby is asleep with little to no eye movement, regular breathing. 2. The baby is asleep with rapid eye movement, irregular breathing. 3. The baby is awake, looking intently at an object, irregular breathing. 4. The baby is awake, placing hands in the mouth, irregular breathing

3 : The test taker could make an educated guess regarding this question even if the term "en face" were unfamiliar. The expression means "face to face," which is clearly implied by the term. Because bonding between parent and child is so important, whenever a baby exhibits the quiet alert behavior, the nurse should encourage the interaction. Although the father may bond with a sleeping baby who is in the en face position, the baby is unable to interact or bond with his or her parent.

48. Which of the following client responses indicates that the nurse's teaching about care following chorionic villus sampling (CVS) has been successful? 1. If the baby stops moving, the woman should immediately go to the hospital. 2. The woman should take oral terbutaline every 2 hours for the next day. 3. If the woman starts to bleed or to contract, she should call her physician. 4. The woman should stay on complete bed rest for the next 48 hours.

3 : The test taker, if familiar with normal pregnancy changes, can immediately eliminate choice 1 since CVS is performed between 10 and 12 weeks' gestation and quickening rarely occurs before 16 weeks' gestation, even in multiparous pregnancies. Spontaneous abortion is the most common complication of CVS; therefore, the woman should report any bleeding or contractions.

8. The parent of a newborn angrily asks the nurse, "Why would the doctor want to give my baby the vaccination for hepatitis B? It's a sexually transmitted disease, you know!" Which of the following is the best response by the nurse? 1. "The hepatitis B vaccine is given to all babies. It is given because many babies get infected from their mothers during pregnancy." 2. "It is important for your baby to get the vaccine in the hospital because the shot may not be available when your child gets older." 3. "Hepatitis B can be a life-threatening infection that is contracted by contact with contaminated blood as well as sexually." 4. "Most parents want to protect their children from as many serious diseases as possible. Hepatitis B is one of those diseases."

3 A number of individuals who contract the hepatitis B virus become long-term carriers of the disease and are able to transmit it to others. They are also at high risk for the development of chronic liver disease and liver cancer

74. On vaginal examination, it is noted that a woman with a well-functioning epidural is in the second stage of labor. The station is −2 and the baseline fetal heart rate is 130 with no decelerations. Which of the following nursing actions is appropriate at this time? 1. Coach the woman to hold her breath while pushing 3 to 4 times with each contraction. 2. Administer oxygen via face mask at 8 to 10 liters per minute. 3. Delay pushing until the baby descends further and the mother has a strong urge to push. 4. Place the woman on her side and assess her oxygen saturation.

3 Although the use of an epidural is not high risk, there can be injuries to the maternal birth canal and/or the fetus when pushing is performed by a mother with no feeling. It is recommended that women who have lost all feeling because of an epidural "labor down," or rest and wait, until the urge to push returns. Once that happens, the woman should perform open glottal pushing.

59. A client who has been taking birth control pills for 2 months calls the clinic with the following complaint: "I have had a bad headache for the past couple of days and I now have pain in my right leg." Which of the following responses should the nurse make? 1. "Continue the pill, but take one aspirin tablet with it each day from now on." 2. "Stop taking the pill, and start using a condom for contraception." 3. "Come to the clinic this afternoon so that we can see what is going on." 4. "Those are common side effects that should disappear in a month or so."

3 Clients who use hormonally based contraceptive methods are high risk for clot formation. This client is communicating symptoms that may indicate the presence of a clot. She should be seen by her practitioner to rule out deep vein thrombosis and a possible stroke.

58. A woman is in active labor and is being monitored electronically. She has just received Stadol 2 mg IM for pain. Which of the following fetal heart responses would the nurse expect to see on the internal monitor tracing? 1. Variable decelerations. 2. Late decelerations. 3. Decreased variability. 4. Transient accelerations

3 It is important for the test taker to remember the side effects of commonly used medications. The analgesics used in labor are opiates. The CNS-depressant effect of the opiates is therapeutic for the mother who is in pain, but the baby is also affected by the medication, often exhibiting decreased variability

5. While performing Leopold's maneuvers on a woman in labor, the nurse palpates a hard round mass in the fundal area, a flat surface on the left side, small objects on the right side, and a soft round mass just above the symphysis. Which of the following is a reasonable conclusion by the nurse? 1. The fetal position is transverse. 2. The fetal presentation is vertex. 3. The fetal lie is vertical. 4. The fetal attitude is flexed.

3 Many obstetric assessments have a component that is sensual and a component that is an interpretation or concept. Leopold's maneuvers are good examples. The nurse palpates specific areas of the pregnant abdomen, but then must interpret or translate what he or she is feeling into a concept. For example, in the scenario presented, the nurse palpates a hard, round mass in the fundal area of the uterus and must interpret that feeling as the fetal head. Similarly, the nurse palpates a soft round mass above the symphysis and must interpret that feeling as the fetal buttocks. With these findings and interpretations, the nurse will then realize that the fetal lie is vertical.

28. A client who is sexually active is asking the nurse about Gardasil, one of the vaccines that is given to prevent human papillomavirus (HPV). Which of the following should be included in the counseling session? 1. Gardasil is not recommended for women who are already sexually active. 2. Gardasil protects recipients from all strains of the virus. 3. The most common side effect from the vaccine is pain at the injection site. 4. Anyone who is allergic to eggs is advised against receiving the vaccine.

3 The CDC Advisory Committee on Immunization Practices recommends that the HPV vaccine— Gardasil—be given to all young men and women between the ages of 11 and 12, or as young as age 9, and up to age 26. There is also a second HPV vaccine on the market, Cervarix. Both Gardasil and Cervarix effectively protect recipients against HPV types 16 and 18—the two types that cause most HPV-related cancers. Only Gardasil®, however, also protects against two additional strains of HPV—types 6 and 11—that cause most cases of genital warts

15. A nurse is teaching a mother how to care for her 3-day-old son's circumcised penis. Which of the following actions demonstrates that the mother has learned the information? 1. The mother cleanses the glans with a cotton swab dipped in hydrogen peroxide. 2. The mother covers the glans with antifungal ointment after rinsing off any discharge. 3. The mother squeezes soapy water from the wash cloth over the glans. 4. The mother replaces the dry sterile dressing before putting on the diaper.

3 The circumcised penis has undergone a surgical procedure, but to apply a dry dressing is potentially injurious. If the dressing adheres to the newly circumcised penis, the incision could bleed. The test taker should be aware that with routine cleaning, as cited above, circumcisions usually heal quickly and rarely become infected.

2. A 19-year-old client with multiple sex partners is being counseled about the hepatitis B vaccination. During the counseling sessions, which of the following should the nurse advise the client to receive? 1. Hepatitis B immune globulin before receiving the vaccine. 2. Vaccine booster every 10 years. 3. Complete series of three intramuscular injections. 4. Vaccine as soon as she becomes 21.

3 The current recommendation by the Centers for Disease Control and Prevention (CDC) is that the hepatitis B vaccine series be administered during the neonatal period. For those who have not received the vaccine in infancy, it can be administered at any age. The second and third shots are administered 1 month and 6 months after the first, respectively

48. A breastfeeding woman is requesting that she be prescribed Seasonale (ethinyl estradiol and levonorgestrel) as a birth control method. Which of the following information should be included in the patient teaching session? 1. The woman will menstruate every 8 to 9 weeks. 2. The pills are taken for 3 out of every 4 weeks. 3. Breakthrough bleeding is a common side effect. 4. Breastfeeding is compatible with the medication.

3 Women who wish to breastfeed can take some types of birth control pills (BCPs), but not pills that contain an estrogen medication. Seasonale contains an estrogen. Estrogen inhibits milk production. If they wish to take BCPs, breastfeeding women should take progestin-only pills.

90. A nurse is caring for a client who states that she is a lesbian. Which of the following should the nurse consider when caring for this client? 1. Lesbian women are usually less sexually active than straight women. 2. Lesbian women need not be asked about domestic violence issues. 3. Lesbian women should be tested for cervical cancer every three to seven years. 4. Lesbian women are at higher risk for bacterial vaginosis than are straight women.

4 : Nurses must be prepared to care for clients in every walk of life. The special needs of gay men and women are often ignored by health care workers. When caring for clients, one question that should be asked is the client's sexual preference. Unless the nurse asks the question, important issues may be missed.

1. The nurse in a pediatric clinic is caring for a 9-year-old girl who has been diagnosed with gonorrhea. Which of the following actions is appropriate for the nurse to take? 1. Notify the physician so the child can be admitted to the hospital. 2. Discuss with the girl the need to stop future sexual encounters. 3. Question the mother about her daughter's menstrual history. 4. Report the girl's medical findings to child protective services.

4 Any time a sexually transmitted disease is discovered in a minor, the nurse has the legal obligation to report the finding to a child protection agency. In addition, if required by law, the health department should also be notified to track and follow up on sexually transmitted infections.

65. A nurse is educating a group of women in her parish about osteoporosis. The nurse should include in her discussion that which of the following is a risk factor for the disease process? 1. Multiparity. 2. Increased body weight. 3. Late onset of menopause. 4. Heavy alcohol intake.

4 Daily consumption of alcohol is a contributing factor to the development of osteoporosis because alcohol interferes with the absorption of vitamin D and calcium in the body. As adequate consumption of the vitamin and mineral is essential for strong bones, alcohol should be consumed in moderation.

86. To decrease the possibility of a perineal laceration during delivery, the nurse performs which of the following interventions prior to the delivery? 1. Assists the woman into a squatting position. 2. Advises the woman to push only when she feels the urge. 3. Encourages the woman to push slowly and steadily. 4. Massages the perineum with mineral oil.

4 During labor, nurses and nurse midwives often massage a woman's perineum to increase the elasticity of the tissue. Because the tissue is more elastic, it is less inclined to tear during the delivery In addition, mothers are often encouraged to begin massaging the tissue during their last trimester.

12. A couple is asking the nurse whether or not their son should be circumcised. On which fact should the nurse's response be based? 1. Boys should be circumcised for them to establish a positive self-image. 2. Boys should not be circumcised because there is no medical rationale for the procedure. 3. Experts from the Centers for Disease Control and Prevention argue that circumcision is desirable. 4. A statement from the American Academy of Pediatrics asserts that circumcision is optional.

4 In this question, authorities were cited—namely, the Centers for Disease Control and Prevention (CDC) and the American Academy of Pediatrics (AAP). The student should be familiar with authorities in the field, including the CDC, AAP, and the Association of Women's Health, Obstetric, and Neonatal Nursing (AWHONN). It is helpful to cite authorities when responding to parents' questions about emotionally charged issues like circumcision.

91. A 2-day-old neonate received a vitamin K injection at birth. Which of the following signs/symptoms in the baby would indicate that the treatment was effective? 1. Skin color is pink. 2. Vital signs are normal. 3. Glucose levels are stable. 4. Blood clots after heel stick

4 It is essential that the test taker be familiar with the actions, normal dosages, recommended routes, and so on of all standard medications administered to the neonate

55. After analyzing an internal fetal monitor tracing, the nurse concludes that there is moderate short-term variability. Which of the following interpretations should the nurse make in relation to this finding? 1. The fetus is becoming hypoxic. 2. The fetus is becoming alkalotic. 3. The fetus is in the middle of a sleep cycle. 4. The fetus has a healthy nervous system.

4 It is important for the test taker to be familiar with situations that can change the fetal heart variability. Normal situations that can decrease the variability include fetal sleep, administration of central nervous system depressant medications, and prematurity. A normal situation that can increase the variability is fetal activity.

6. A woman is menstruating. If hormonal studies were to be done at this time, which of the following hormonal levels would the nurse expect to see? 1. Both estrogen and progesterone are high. 2. Estrogen is high and progesterone is low. 3. Estrogen is low and progesterone is high. 4. Both estrogen and progesterone are low.

4 Menstruation begins the menstrual cycle. Since pregnancy has not occurred, the test taker can deduce that the hormones of pregnancy did not remain elevated. Option 4, that both estrogen and progesterone are low, is, therefore, the correct response.

77. A physician writes in a breastfeeding mother's chart, "Ampicillin 500 mg q 6 h po. Baby should be bottle fed until medication is discontinued." What should be the nurse's next action? 1. Follow the order as written. 2. Call the doctor and question the order. 3. Follow the antibiotic order but ignore the order to bottle feed the baby. 4. Refer to a text to see whether the antibiotic is safe while breastfeeding.

4 Nurses not only are responsible for instituting the orders made by physicians and other primary health care practitioners but also have independent practice for which they are accountable. In this scenario, the nurse is accountable to the client. Because the medication is compatible with breastfeeding, but the physician was apparently unaware of that fact, it is the nurse's responsibility to convey that information to the doctor and to advocate for the client. The NIH has created a Web site— LactMed—where the potential danger of medications during lactation can be checked

A nursing student is examining a client's chart on the antepartum unit and asks why an umbilical artery Doppler flow test is ordered. What would be an appropriate response by the nurse? Select all that apply. "It is used for some mothers to identify the gestational age of the fetus." "It is used to detect any abnormal structures of the fetus." "It is a noninvasive screening technique that uses advanced ultrasound technology to assess resistance to blood flow in the placenta." "Images are obtained of blood flow in the umbilical artery." "This test assesses placental perfusion."

"It is a noninvasive screening technique that uses advanced ultrasound technology to assess resistance to blood flow in the placenta." "Images are obtained of blood flow in the umbilical artery." "This test assesses placental perfusion."

66. A nurse is advising a pregnant woman about the danger signs of pregnancy. The nurse should teach the mother that she should notify the physician immediately if she experiences which of the following signs/symptoms? Select all that apply. 1. Convulsions. 2. Double vision. 3. Epigastric pain. 4. Persistent vomiting. 5. Polyuria.

1 2 3 4 : The danger signs of pregnancy are signs or symptoms that can occur in an otherwise healthy pregnancy that are likely due to serious pregnancy complications. For example, double vision, epigastric pain, and blurred vision are symptoms of the hypertensive illnesses of pregnancy, and persistent vomiting is a symptom of hyperemesis gravidarum.

77. A 39-year-old, 16-week-gravid woman has had an amniocentesis. Before discharge, the nurse teaches the woman to call her doctor if she experiences any of the following side effects? Select all that apply. 1. Fever or chills. 2. Lack of fetal movement. 3. Abdominal pain. 4. Rash or pruritus. 5. Vaginal bleeding.

1 2 3 5 During an amniocentesis, the amniotic sac is entered with a large needle. As a result of the procedure, a number of complications can develop, including infection, preterm labor, rupture of the membranes, and/or fetal injury. Although the incidence of complications is small, it is very important for the nurse to advise the client of the signs of each of these problems.

5. A woman is being interviewed by a triage nurse at a medical doctor's office. Which of the following signs/symptoms by the client would warrant the nurse to suggest that a pregnancy test be done? Select all that apply. 1. Amenorrhea. 2. Fever. 3. Fatigue. 4. Nausea. 5. Dysuria.

1 3 4 This question is easily answered if the test taker is familiar with the presumptive signs of pregnancy—that is, the subjective complaints of pregnancy

8. The labor and delivery nurse performs Leopold's maneuvers. A soft round mass is felt in the fundal region. A flat object is noted on the left and small objects are noted on the right of the uterus. A hard round mass is noted above the symphysis. Which of the following positions is consistent with these findings? 1. Left occipital anterior (LOA). 2. Left sacral posterior (LSP). 3. Right mentum anterior (RMA). 4. Right sacral posterior (RSP).

1 The test taker must review fetal positioning. This is an especially difficult concept to understand. The best way to learn the three-dimensional concept of fetal position is to look at the pictures in a text and then to get a doll and to imitate the pictures by placing the doll into each of the positions.

9. When a nurse is teaching a woman about her menstrual cycle, which of the following is the most important change that happens during the follicular phase of the menstrual cycle? 1. Maturation of the graafian follicle. 2. Multiplication of the fimbriae. 3. Secretion of human chorionic gonadotropin. 4. Proliferation of the endometrium.

1 The test taker should always try to find a clue in the stem for the answer to the question. In this question, the test taker is requested to choose "the most important change that happens during the follicular phase." In general, things are named logically, and this case is no different. The follicular phase is the period of the menstrual cycle when the follicle matures.

98. The nursing management of a neonate with physiological jaundice should be directed toward which of the following client care goals? 1. The baby will exhibit no signs of kernicterus. 2. The baby will not develop erythroblastosis fetalis. 3. The baby will have a bilirubin of 16 mg/dL or higher at discharge. 4. The baby will spend at least 20 hours per day under phototherapy.

1 This question asks the test taker to identify a client care goal for a newborn with physiological jaundice. The client care goal reflects the nurse's desired patient care outcome. The development of kernicterus is a potential pathological outcome resulting from hyperbilirubinemia. The client care goal, therefore, is that the neonate not develop kernicterus.

6. A breastfeeding woman has been counseled on how to prevent engorgement. Which of the following actions by the mother shows that the teaching was effective? 1. She pumps her breasts after each feeding. 2. She feeds her baby every 2 to 3 hours. 3. She feeds her baby 10 minutes on each side. 4. She supplements each feeding with formula.

2 This question is similar to the preceding question except that this question tests the nurse's ability to evaluate a client's response rather than to perform a nursing action.

93. A mother questions why the ophthalmic medication is given to the baby. Which of the following responses by the nurse would be appropriate to make at this time? 1. "I am required by law to give the medicine." 2. "The medicine helps to prevent eye infections." 3. "The medicine promotes neonatal health." 4. "All babies receive the medicine at delivery."

2 When asked a direct question by a client, it is important for the nurse to give as complete a response as possible. Trite responses like "All babies receive the medication at birth" do not provide information to the client. It is the right of all clients to receive accurate and complete information about their own treatments and, because the neonate is a dependent, the parents have the right to receive accurate and complete information about their baby's treatments.

4. The nurse is administering Depo-Provera (medroxyprogesterone acetate) to a postpartum client. Which of the following data must the nurse consider before administering the medication? 1. The capsule must be taken at the same time each day. 2. The client must be taught to use sunscreen whenever in the sunlight. 3. The medicine is contraindicated if the woman has lung or esophageal cancer. 4. The client must use an alternate form of birth control for the first two months.

2 Women can develop dark patches on their skin when using Depo-Provera. The patches often become darker in women who are in the sun without protection. It is strongly recommended that women who use Depo-Provera use sunscreen whenever they are exposed to the sun

105. The nurse is assessing the Bishop score on a postdates client. Which of the following measurements will the nurse assess? Select all that apply. 1. Gestational age. 2. Rupture of membranes. 3. Cervical dilation. 4. Fetal station. 5. Cervical position.

3 4 5 The Bishop score is calculated to determine the inducibility of the cervix. Although gestational age may be an indication for calculating the score, it does not have a direct impact on the status of the cervix. Similarly, although rupture of the membranes may be an indication for calculating the score, that fact does not have a direct impact on the status of the cervix.

13. A 29-week-gestation woman diagnosed with severe preeclampsia is noted to have blood pressure of 170/112, 4+ proteinuria, and a weight gain of 10 pounds over the past 2 days. Which of the following signs/symptoms would the nurse also expect to see? 1. Fundal height of 32 cm. 2. Papilledema. 3. Patellar reflexes of +2. 4. Nystagmus.

3 Intracranial pressure (ICP) is present in a client with severe preeclampsia because she is third spacing large quantities of fluid. As a result of the ICP, the optic disk swells and papilledema is seen when the disk is viewed through an ophthalmoscope

32. The nurse is about to elicit the Moro reflex. Which of the following responses should the nurse expect to see? 1. When the cheek of the baby is touched, the newborn turns toward the side that is touched. 2. When the lateral aspect of the sole of the baby's foot is stroked, the toes extend and fan outward. 3. When the baby is suddenly lowered or startled, the neonate's arms straighten outward and the knees flex. 4. When the newborn is supine and the head is turned to one side, the arm on that same side extends.

3 The test taker must be familiar not only with the reason for eliciting reflexes but also with the correct technique for eliciting the actions.

18. Which of the following sexually transmitted infections is characterized by a foulsmelling, yellow-green discharge that is often accompanied by vaginal pain and dyspareunia? 1. Syphilis. 2. Herpes simplex. 3. Trichomoniasis. 4. Condylomata acuminata.

3 Trichomoniasis is a sexually transmitted infection caused by a protozoan. Women who develop the infection during pregnancy may develop preterm labor. Women who are infected with trichomoniasis have an increased risk of contracting HIV if exposed

98. A client is receiving terbutaline (Brethine) for preterm labor. Which of the following findings would warrant stopping the infusion? Select all that apply. 1. Change in contraction pattern from q 3 min × 90 sec to q 2 min × 60 sec. 2. Change in fetal heart pattern from no decelerations to early decelerations. 3. Change in beat-to-beat variability from minimal to moderate. 4. Change in fetal heart rate from 160 bpm to 210 bpm. 5. Change in the amniotic sac from intact to ruptured.

4 5 Terbutaline, a beta agonist, stimulates the "fight or flight" response in the mother and in the fetus. The fetal heart rate, therefore, increases in response to the medication. When the rate is too high, however, there is insufficient time for the blood to enter the heart, which leads to a drop in cardiac output.

114. A client has been admitted with a diagnosis of hyperemesis gravidarum. Which of the following lab values would be consistent with this diagnosis? 1. pO2 90, pCO2 35, HCO3 19 mEq/L, pH 7.30. 2. pO2 100, pCO2 30, HCO3 21 mEq/L, pH 7.50. 3. pO2 60, pCO2 50, HCO3 28 mEq/L, pH 7.30. 4. pO2 90, pCO2 45, HCO3 30 mEq/L, pH 7.50.

4 The test taker must not panic when confronted with blood gas data. If assessed methodically, the test taker should have little trouble determining the correct answer. The first action is to determine what the results should show. If a woman is vomiting repeatedly, one would expect her to have lost acid from the stomach. She would, therefore, be in metabolic alkalosis. The test taker should then look at the pH levels—they should be elevated—and the O2 levels— they should be normal—to begin to determine which response is correct.

27. Which finding should the nurse expect when assessing a client with placenta previa? 1. Severe occipital headache. 2. History of thyroid cancer. 3. Previous premature delivery. 4. Painless vaginal bleeding.

4 There are three different forms of placenta previa: low-lying placenta—one that lies adjacent to, but not over, the internal cervical os; partial— one that partially covers the internal cervical os; and complete—a placenta that completely covers the internal cervical os. There is no way to deliver a live baby vaginally when a client has a complete previa, although there are cases when live babies have been delivered when the clients had low-lying or partial previas.

The labor and delivery (L&D) educator has explained to a group of nurses how the uteroplacental unit functions. Which components are necessary for the fetus to receive appropriate oxygenation? Select all that apply. Adequate oxygenation of the mother Adequate uteroplacental circulation Adequate umbilical circulation Adequate fetal blood volume Adequate blood flow to the placenta

Adequate oxygenation of the mother Adequate uteroplacental circulation Adequate umbilical circulation Adequate blood flow to the placenta

While caring for a client, which interventions would the nurse include in the nursing care plan to provide culturally competent care? Select all that apply. Describe hospital protocols that will be followed during the delivery. Provide teaching on nonpharmacological pain management options as they are preferred by women of the client's culture. Ask the client if they have any religious or cultural preferences they would like followed during their labor. Ask the client what gender pronouns they prefer and use them. Determine who the client's support person is and how they will participate in their care.

Ask the client if they have any religious or cultural preferences they would like followed during their labor. Ask the client what gender pronouns they prefer and use them. Determine who the client's support person is and how they will participate in their care.

The nurse is reviewing the biophysical profile (BPP) results and would expect which variables to be included in this test? Select all that apply. Fetal position Fetal tone Amniotic fluid volume Fetal breathing movements Fetal movement

Fetal tone Amniotic fluid volume Fetal breathing movements Fetal movement

Which assessments of uterine activity are obtained by the nurse when the client has an intrauterine pressure catheter (IUPC) placed? Select all that apply. Frequency Intensity Duration Fetal Heart Rate Resting Tone

Frequency Intensity Duration Resting Tone

The labor and delivery (L&D) unit educator discusses the prioritization of fetal monitoring goals, creating a plan of care, and setting goals for implementation with a group of nurses. Which goals are correct? Select all that apply. Interpret ongoing assessment of fetal oxygenation. Prevent significant fetal academia. Minimize unnecessary interventions. Promote a satisfying family-centered birth experience. Use of electronic fetal monitors (EFM) for all laboring women in the United States is required.

Interpret ongoing assessment of fetal oxygenation. Prevent significant fetal academia. Minimize unnecessary interventions. Promote a satisfying family-centered birth experience.

The instructor is teaching the modes of fetal heart rate (FHR) and contraction assessment to a class of nursing students. What are the modes of uterine monitoring? Select all that Apply Auscultation Palpation Fetal Spiral Electrode Intrauterine Pressure Catheter Tocodynamometer

Palpation Intrauterine Pressure Catheter Tocodynamometer

The nurse is caring for a gravid patient who is carrying twins. Which complications would the nurse monitor the patient for? Select all that apply. Preeclampsia Gestational Diabetes Abruptio Placentae Sickle-cell anemia Cardiomyopathy

Preeclampsia Gestational Diabetes Abruptio Placenta Cardiomyopathy

The nurse is assessing a laboring client. Which signs and symptoms does the nurse recognize indicate movement into the second stage of labor? Select all that apply. Intense contractions every 4-5 minutes Significant Bloody Show Stated claim by client of needing a bowel movement Noted Trembling of client Vomiting

Significant Bloody Show Stated claim by client of needing a bowel movement Noted Trembling of client Vomiting

The nurse is describing baseline fetal heart rate (FHR) to a practicum student. What would the nurse include when teaching abo"Episodic changes in baseline of FHR occur independent of uterine contractions (UCs)." "FHR is rounded to increments of 5 beats per minute during a 10-minute window. This must be at least 2 minutes of identifiable baseline segment."ut the definition and assessment criteria related to baseline FHR? Select all that apply. "Periodic changes in baseline of FHR occur in relation to uterine contractions." "Recurrent changes in baseline of FHR occur in less than 50% of the contractions in 20 minutes." "Intermittent changes in baseline of FHR occur in greater than 50% of the contractions in 20 minutes." "Episodic changes in baseline of FHR occur independent of uterine contractions (UCs)." "FHR is rounded to increments of 5 beats per minute during a 10-minute window. This must be at least 2 minutes of identifiable baseline segment."

"Periodic changes in baseline of FHR occur in relation to uterine contractions." "Episodic changes in baseline of FHR occur independent of uterine contractions (UCs)." "FHR is rounded to increments of 5 beats per minute during a 10-minute window. This must be at least 2 minutes of identifiable baseline segment."

36. A woman is to receive methotrexate IM for an ectopic pregnancy. The nurse should teach the woman about which of the following common side effects of the therapy? Select all that apply. 1. Nausea and vomiting. 2. Abdominal pain. 3. Fatigue. 4. Light-headedness. 5. Breast tenderness.

1 2 3 4 Because methotrexate is an antineoplastic agent, the nurse would expect to see the same types of complaints that he or she would see in a patient receiving chemotherapy for cancer. It is very important that the abdominal pain seen with the medication not be dismissed because the most common complaint of women with ectopic pregnancies is pain. The source of the pain, therefore, must be clearly identified

33. A woman has decided to hire a doula to work with her during labor and delivery. Which of the following actions would be appropriate for the doula to perform? Select all that apply. 1. Give the woman a back rub. 2. Assist the woman with her breathing. 3. Assess the fetal heart rate. 4. Check the woman's blood pressure. 5. Regulate the woman's intravenous.

1 2 Even if the test taker were unfamiliar with the role of the doula, he or she could deduce the answers to this question. Three of the responses involve physiological assessments or interventions while two of the responses deal with providing supportive care, the role of the doula.

13. On examination, it is noted that a full-term primipara in active labor is right occipitoanterior (ROA), 7 cm dilated, and +3 station. Which of the following should the nurse report to the physician? 1. Descent is progressing well. 2. Fetal head is not yet engaged. 3. Vaginal delivery is imminent. 4. External rotation is complete.

1 This question includes a number of concepts. Descent and station are discussed in answer options 1 and 2. The dilation of the cervix, which is related to the fact that the woman is a primigravida, is discussed in choice 3. And, one of the cardinal moves of labor— external rotation—is included in choice 4. The test taker must be prepared to answer questions that are complex and that include diverse information. In a 7 cm dilated primipara, with a baby at +3 station, vaginal delivery is not imminent, but the fetal head is well past engagement and descent is progressing well. External rotation has not yet occurred because the baby's head has not yet been birthed.

24. Which of the following long-term goals is appropriate for a client, 10 weeks' gestation, who is diagnosed with gestational trophoblastic disease (hydatidiform mole)? 1. Client will be cancer free 1 year from diagnosis. 2. Client will deliver her baby at full term without complications. 3. Client will be pain free 3 months after diagnosis. 4. Client will have normal hemoglobin and hematocrit at delivery

1 When nurses plan care, they have in mind short-term and longterm goals that their clients will achieve. Short-term goals usually have a time frame of a week or two and often are specific to the client's current hospitalization. Long-term goals are expectations of client achievement over extended periods of time. It is important for nurses to develop goals to implement appropriate nursing interventions.

85. A nurse is about to inject RhoGAM into an Rh-negative mother. Which of the following is the preferred site for the injection? 1. Deltoid. 2. Dorsogluteal. 3. Vastus lateralis. 4. Ventrogluteal.

1 Whenever possible, it is preferable to inject the antibodies into the recommended injection site. The antibodies are absorbed optimally from that site and, therefore, are more apt to suppress the mother's immune response.

93. A nurse has provided a young woman with preconception counseling. Which of the statements by the woman indicates that the teaching was successful? Select all that apply. 1. "As soon as I think I may be pregnant, I should stop drinking alcohol." 2. "It is important for me to see my medical doctor for a complete physical." 3. "I should make sure that my daily multivitamin contains folic acid." 4. "When I go to my dentist for a checkup I should state that I may be pregnant." 5. "From now until I deliver I should refrain from eating sushi and rare meat."

2 3 4 5 Embryogenesis often occurs before a woman is aware that she is pregnant. Because teratogenic insults can injure the developing embryo, it is essential that women plan their pregnancies and avoid teratogens when attempting to become pregnant.

3. A nurse is explaining to a client about monthly hormonal changes. Starting with day 1 of the menstrual cycle, please place the following four hormones in the chronological order in which they elevate during the menstrual cycle. 1. Follicle-stimulating hormone. 2. Gonadotropin-releasing hormone. 3. Luteinizing hormone. 4. Progesterone.

2 1 3 4 Gonadotropin-releasing hormone stimulates the production of folliclestimulating hormone (FSH) and lutenizing hormone (LH). FSH rises first and LH follows. After ovulation, progesterone rises

81. A woman who has been abused for a number of years is finally seeking assistance in leaving her relationship. Identify the actions that the nurse should take at this time. Select all that apply. 1. Comment that the victim could have left long ago. 2. Assist the victim to develop a safety plan. 3. Remind the victim that the abuse was not her fault. 4. Assure the victim that she will receive support for her decision. 5. Help the victim to contact a domestic violence center.

2 3 4 5 After many years of abuse, victims often have very low selfesteem and are very frightened of their abusers. They need a great deal of emotional support as well as clear, structured guidance in how to leave the relationship. Nurses must be prepared to supply the support

6. A 24-week-gravid client is being seen in the prenatal clinic. She states, "I have had a terrible headache for the past 2 days." Which of the following is the most appropriate action for the nurse to perform next? 1. Inquire whether or not the client has allergies. 2. Take the woman's blood pressure. 3. Assess the woman's fundal height. 4. Ask the woman about stressors at work.

2 Headache is a symptom of preeclampsia. Preeclampsia, a serious complication, is a hypertensive disease of pregnancy. To determine whether or not the client is preeclamptic, the next action by the nurse would be to assess the woman's blood pressure.

77. Immediately following administration of an epidural anesthesia, the nurse must monitor the mother for which of the following? 1. Paresthesias in her feet and legs. 2. Drop in blood pressure. 3. Increase in central venous pressure. 4. Fetal heart accelerations.

2 The test taker must be familiar with the side effects of all medications. If no other therapeutic interventions are performed, virtually all women will show signs of hypotension after epidural administration. The change is related to two phenomena: dilation of the vessels in the pelvis and increased compression of the vena cava.

31. A nurse has just performed a vaginal examination on a client in labor. The nurse palpates the baby's buttocks as facing the mother's right side. Where should the nurse place the external fetal monitor electrode? 1. Left upper quadrant (LUQ). 2. Left lower quadrant (LLQ). 3. Right upper quadrant (RUQ). 4. Right lower quadrant (RLQ).

3 Although the question does not tell the test taker whether the sacrum is facing anteriorly or posteriorly, it does provide the information that the sacrum is felt toward the mother's right. Because this baby is in the sacral presentation and the back is toward the right, the best location for the fetal monitor is in the RUQ, at the level of the fetal back.

70. A medication order reads: Methergine (ergonovine) 0.2 mg po q 6 h × 4 doses. Which of the following assessments should be made before administering each dose of this medication? 1. Apical pulse. 2. Lochia flow. 3. Blood pressure. 4. Episiotomy.

3 Methergine is an oxytoxic agent that works directly on the myofibrils of the uterus. The smooth muscle of the vascular tree is also affected. The blood pressure may elevate, therefore, to dangerous levels. The medication should be held if the blood pressure is 130/90 or higher and the woman's health care practitioner should be notified if appropriate

18. Four women request to labor in the hospital bathtub. In which of the following situations is the procedure contraindicated? Select all that apply. 1. Woman during transition. 2. Woman during second stage of labor. 3. Woman receiving oxytocin for induction. 4. Woman with meconium-stained fluid. 5. Woman with fetus in the occiput posterior position.

3 4 Hydrotherapy is an excellent complementary therapy for the laboring woman. The warm water is relaxing and many women find that their pain is minimized. Induction and continuous electronic fetal monitoring, however, are incompatible with the intervention.

85. Which of the following situations in a fully dilated client is incompatible with a forceps delivery? Select all that apply. 1. Maternal history of asthma. 2. Right occiput posterior position at +4 station. 3. Transverse fetal lie. 4. Fetal heart rate of 60 beats per minute at -1 station. 5. Maternal history of cerebral palsy.

3 4 It is unsafe to use forceps to deliver a baby when the baby's station is above +2. When the baby is above that station, it is unknown whether or not there is sufficient room in the pelvis for the baby to pass. If there should be too little space, very serious fetal complications could arise, including fractured skull and subdural hematoma.

93. A nurse is caring for a gravid client who is G1 P0000, 35 weeks' gestation. Which of the following would warrant the nurse to notify the woman's health care practitioner that the client is in preterm labor? Select all that apply. 1. Contraction frequency every 15 minutes. 2. Effacement 10%. 3. Dilation 3 cm. 4. Cervical length of 2 cm. 5. Contraction duration of 30 seconds.

3 4 Preterm labor is defined as labor before 38 weeks' gestation with 3 or more contractions occurring within a 30-minute period PLUS cervical change of one of the following: cervical effacement greater than 80%, cervical dilation greater than 1 cm, or cervical length of less than 2.5 cm. The change in cervical length is diagnosed by transvaginal ultrasound.

21. The public health nurse calls a woman and states, "I am afraid that I have some disturbing news. A man who has been treated for gonorrhea by the health department has told them that he had intercourse with you. It is very important that you seek medical attention." The woman replies, "There is no reason for me to go to the doctor! I feel fine!" Which of the following replies by the nurse is appropriate at this time? 1. "I am sure that you are upset by the disturbing news, but there is no reason to be angry with me." 2. "I am sorry. We must have received the wrong information." 3. "That certainly could be the case. Women often report no symptoms." 4. "All right, but please tell me your contacts because it is possible for you to pass the disease on even if you have no symptoms."

3 This client is exhibiting signs of denial. The nurse must empathize with the woman regarding the unexpected and unwanted news, but the nurse also must convince the woman to seek care. Giving her the information that many women have no signs of symptoms of disease is essential.

3. A postpartum client has decided to use Depo-Provera (medroxyprogesterone acetate) as her contraceptive method. What should the nurse advise the client regarding this medication? 1. Take the pill at the same time each day. 2. Refrain from breastfeeding while using the method. 3. Expect to have no periods as long as she takes the medicine. 4. Consider switching to another birth control method in a year or so.

4 There is a black box recommendation on the Depo-Provera label. A black box warning is placed on some prescription medications that have been found to have significant side effects. The Food and Drug Administration (FDA) has the power to require pharmaceutical companies to include a black box on a medication that, although approved for use, carries risks when taken. In the case of Depo-Provera, there is an increased risk of osteoporosis.

32. A woman, 8 weeks pregnant, is admitted to the obstetric unit with a diagnosis of threatened abortion. Which of the following tests would help to determine whether the woman is carrying a viable or a nonviable pregnancy? 1. Luteinizing hormone level. 2. Endometrial biopsy. 3. Hysterosalpinogram. 4. Serum progesterone level

4 When a previously gravid client is seen by her health care practitioner with a complaint of vaginal bleeding, it is very important to determine the viability of the pregnancy as soon as possible. Situational crises are often exacerbated when clients face the unknown. One relatively easy way to determine the viability of the conceptus is by performing a serum progesterone test; high levels indicate a viable baby whereas low levels indicate a pregnancy loss. Ultrasonography to assess for a beating heart may also be performed.

The nurse has just completed a fetal monitoring course and is explaining the normal findings of structured intermittent auscultation (SIA) with a handheld Doppler. Which would the nurse identify as a normal finding of SIA? Select all that apply. Moderate variability Normal baseline between 110 to 160 bpm Regular rhythm Presence of fetal heart rate (FHR) increases from baseline Absence of fetal heart rate (FHR) decreases from baseline

Normal baseline between 110 to 160 bpm Regular rhythm Presence of fetal heart rate (FHR) increases from baseline Absence of fetal heart rate (FHR) decreases from baseline

The nurse is teaching a group of students about the different types of tests performed at different stages of pregnancy. Which statement would the nurse include in the teaching? Select all that apply. "Amniocentesis is an ultrasound assessment of the fetal status with a nonstress test (NST). It utilizes ultrasound with an electronic fetal monitor (EFM) to assess five variables." "Biophysical profile (BPP) is a diagnostic procedure where a needle is inserted through the abdominal wall into the uterine cavity to obtain amniotic fluid and is commonly performed for genetic testing." "Amniocentesis is necessary to elevate the uterus out of the pelvis for better visualization of the fetus's body. It is done during the first half of the pregnancy." "A nonstress test (NST) is a screening tool that uses fetal heart rate (FHR) patterns and accelerations as an indicator of fetal well-being." "Maternal serum alpha-fetoprotein is a screening test that assesses for levels of alpha-fetoprotein (AFP) in the maternal blood to detect certain developmental defects in the fetus, such as neural tube defects (NTDs)."

"A nonstress test (NST) is a screening tool that uses fetal heart rate (FHR) patterns and accelerations as an indicator of fetal well-being." "Maternal serum alpha-fetoprotein is a screening test that assesses for levels of alpha-fetoprotein (AFP) in the maternal blood to detect certain developmental defects in the fetus, such as neural tube defects (NTDs)."

18. The nurse is providing counseling to a group of sexually active single women. Most of the women have expressed a desire to have children in the future, but not within the next few years. Which of the following actions should the nurse suggest the women take to protect their fertility for the future? Select all that apply. 1. Use condoms during intercourse. 2. Refrain from smoking cigarettes. 3. Maintain an appropriate weight for height. 4. Exercise in moderation. 5. Refrain from drinking carbonated beverages.

1 2 3 4 There are a number of factors that can affect fertility. Some of the factors are beyond a woman's control. For example, a woman may not marry until she is in her 30s and, consequently, may delay conception. Other factors, such as smoking cigarettes and exercising, are controllable.

65. A nurse administered RhoGAM to a client whose blood type is A+ (positive). Which of the following responses would the nurse expect to see? Select all that apply. 1. Fever. 2. Flank pain. 3. Dark-colored urine. 4. Swelling at the injection site. 5. Polycythemia.

1 2 3 When RhoGAM is administered to an Rh+ (positive) client, antibodies against the client's red blood cells are being injected into her body. A hemolytic response similar to one seen when a client receives the wrong type of blood may develop.

61. A nurse who is caring for a pregnant diabetic should carefully monitor the client for which of the following? Select all that apply. 1. Urinary tract infection. 2. Multiple gestation. 3. Metabolic acidosis. 4. Pathological hypotension. 5. Hypolipidemia

1 3 It is very important for the test taker to read each response carefully. If the test taker were to read the responses to the preceding question very quickly, he or she might choose incorrect answers. For example, the test taker might pick pathological hypotension, assuming that it says "hypertension." Pregnant diabetics are high risk for UTIs because they often excrete glucose in their urine. The glucose is an excellent medium for bacterial growth. They also should be assessed carefully for acidosis because an acidotic environment can be life threatening to a fetus.

57. A mother is told that she should bottle feed her child for medical reasons. Which of the following maternal disease states are consistent with the recommendation? Select all that apply. 1. Untreated, active tuberculosis. 2. Hepatitis B surface antigen positive. 3. Human immunodeficiency virus positive. 4. Chorioamnionitis. 5. Mastitis.

1 3 The test taker should remember that there are very few instances when breastfeeding is contraindicated. Mothers who are hepatitis B positive may breastfeed because it has not been shown that transmission rates increase with breastfeeding

116. An ultrasound has identified that a client's pregnancy is complicated by oligohydramnios. The nurse would expect that an ultrasound may show that the baby has which of the following structural defects? 1. Dysplastic kidneys. 2. Coarctation of the aorta. 3. Hydrocephalus. 4. Hepatic cirrhosis.

1 The majority of amniotic fluid is produced by the fetal kidneys. When a pregnancy is complicated by oligohydramnios, ultrasounds may be performed to check for defects in the fetal renal system.

74. The nurse has provided teaching to a post-op cesarean client who is being discharged on Colace (docusate sodium) 100 mg po tid. Which of the following would indicate that the teaching was successful? 1. The woman swallows the tablets whole. 2. The woman takes the pills between meals. 3. The woman calls the doctor if she develops a headache. 4. The woman understands that her urine may turn orange.

1 The medication, a stool softener, is contained in a capsule that must be swallowed whole. Many physicians order Colace for postoperative cesarean section clients until their bowel patterns return to normal.

13. A woman is noted to have multiple soft warts on her perineum and rectal areas. The nurse suspects that this client is infected with which of the following sexually transmitted infections? 1. Human papillomavirus (HPV). 2. Human immunodeficiency virus (HIV). 3. Syphilis. 4. Trichomoniasis.

1 The nurse should be familiar with the primary symptoms of sexually transmitted infections. A woman may confide in the nurse about symptoms that she is experiencing. The nurse must be able to determine when symptoms require medical attention.

89. An infant of a diabetic mother, 40 weeks' gestation, weight 4,500 grams, has just been admitted to the neonatal nursery. The neonatal intensive care nurse will monitor this baby for which of the following? Select all that apply. 1. Hyperreflexia. 2. Hypoglycemia. 3. Respiratory distress. 4. Opisthotonus. 5. Nuchal rigidity.

2 3 Babies of diabetic mothers are at high risk for respiratory distress because their lung fields develop more slowly than the lung fields of babies of normoglycemic mothers. Even full-term infants of diabetic mothers sometimes have respiratory difficulties.

26. Which of the following findings would the nurse expect to see when assessing a firsttrimester gravida suspected of having gestational trophoblastic disease (hydatidiform mole) that the nurse would not expect to see when assessing a first-trimester gravida with a normal pregnancy? Select all that apply. 1. Hematocrit 39%. 2. Grape-like clusters passed from the vagina. 3. Markedly elevated blood pressure. 4. White blood cell count 8,000/mm3. 5. Hypertrophied breast tissue.

2 3 It is very important that the test taker know the normal values of common laboratory values, especially the complete blood count, and that the test taker be familiar with deviations from normal diagnostic signs and symptoms.

82. A woman with multiple bruises on her arms and face is seen in the emergency department, accompanied by her partner. When asked about the injuries, the partner states, "She ran into a door." Which of the following actions by the nurse is of highest priority? 1. Take the woman's vital signs. 2. Interview the woman in private. 3. Assess for additional bruising. 4. Document the location of the bruises

2 This client is exhibiting classic signs of physical abuse. The partner is domineering and the client has injuries that are not supported by the history. To obtain a more accurate history, the nurse must interview the client alone. This can often take place in the women's restroom since, unless this is a lesbian relationship, the partner is unable to follow.

103. A baby has just been circumcised. If bleeding occurs, which of the following actions should be taken first? 1. Put the baby's diapers on as tightly as possible. 2. Apply light pressure to the area with sterile gauze. 3. Call the physician who performed the surgery. 4. Assess the baby's heart rate and oxygen saturation.

2 This is a prioritizing question. The nurse's first action must be to provide immediate first aid to best stop the bleeding. Then the nurse must obtain assistance and assess the baby's vital signs to see if they have deviated.

72. The nurse should warn a client who is about to receive Methergine (ergonovine) of which of the following side effects? 1. Headache. 2. Nausea. 3. Cramping. 4. Fatigue.

3 Methergine is administered to postpartum clients to stimulate their uterus to contract. As a consequence, clients frequently complain of cramping after taking the medication. The nurse can administer the prn pain medication to the client at the same time the Methergine is administered to help to mitigate the client's discomfort.

97. A full-term baby's bilirubin level is 12 mg/dL on day 3. Which of the following neonatal behaviors would the nurse expect to see? 1. Excessive crying. 2. Increased appetite. 3. Lethargy. 4. Hyperreflexia.

3 The test taker should be familiar with the normal bilirubin values of the healthy full-term baby (less than 2 mg/dL in cord blood to approximately 12 to 14 mg/dL on days 3 to 5), as well as those values that may result in kernicterus, an infiltration of bilirubin into neural tissue. Brain damage rarely develops when serum bilirubin levels are below 20 mg/dL.

6. A woman, G3 P1010, is receiving oxytocin (Pitocin) via IV pump at 3 milliunits/min. Her current contraction pattern is every 3 minutes × 45 seconds with moderate intensity. The fetal heart rate is 150 to 160 bpm with moderate variability. Which of the following interventions should the nurse take at this time? 1. Stop her infusion. 2. Give her oxygen. 3. Change her position. 4. Monitor her labor.

4 Even if the test taker were unfamiliar with a normal contraction

1. During a prenatal interview, a client tells the nurse, "My mother told me she had toxemia during her pregnancy and almost died!" Which of the following questions should the nurse ask in response to this statement? 1. "Does your mother have a cardiac condition?" 2. "Did your mother tell you what she was toxic from?" 3. "Does your mother have diabetes now?" 4. "Did your mother say whether she had a seizure or not?"

4 The hypertensive illnesses of pregnancy used to be called toxemia of pregnancy as well as pregnancy-induced hypertension. That term is still heard in the community because the mothers and grandmothers of clients were told that they had toxemia of pregnancy. Because daughters of clients who have had preeclampsia are high risk for hypertensive illness, it is important to find out whether or not the client's mother had developed eclampsia.

43. A gravid woman is carrying monochorionic twins. For which of the following complications should this pregnancy be monitored? 1. Oligohydramnios. 2. Placenta previa. 3. Cephalopelvic disproportion. 4. Twin-to-twin transfusion.

4 The key to answering this question is the fact that the twins originate from the same egg—that is, they are monozygotic twins. They share a placenta and a chorion. Because their blood supply is originating from the same source, the twins' circulations are connected. As a result, one twin may become the donor twin while the second twin may become the recipient. The donor grows poorly and develops severe anemia. The recipient becomes polycythemic and large.

34. A client is receiving an epidural infusion of a narcotic for pain relief after a cesarean section. The nurse would report to the anesthesiologist if which of the following were assessed? 1. Respiratory rate 8 rpm. 2. Complaint of thirst. 3. Urinary output of 250 mL/hr. 4. Numbness of feet and ankles.

1 One of the serious complications of narcotic administration is respiratory depression. This client's respiratory rate is well below expected. The nurse should continue to monitor the client carefully and notify the anesthesiologist of the complication.

10. A nurse is monitoring the labor of a client who is receiving IV oxytocin (Pitocin) at 6 mL per hour. Which of the following clinical signs would lead the nurse to stop the infusion? 1. Change in maternal pulse rate from 76 to 98 bpm. 2. Change in fetal heart rate from 128 to 102 bpm. 3. Maternal blood pressure of 150/100. 4. Maternal temperature of 102.4°F.

2 The test taker must determine which of the vital signs is unsafe in the presence of oxytocin. Oxytocin increases the contractility of the uterine muscle. When the muscle contracts, the blood supply to the fetus is diminished. A drop in fetal heart rate, therefore, is indicative of poor oxygenation to the fetus and is unsafe in the presence of oxytocin.

22. In addition to breathing with contractions, which of the following actions can help a woman in the first stage of labor to work with her pain? 1. Lying in the lithotomy position. 2. Performing effleurage. 3. Practicing Kegel exercises. 4. Pushing with each contraction.

2 There are a number of actions that mothers can take that can support their breathing during labor. Walking, swaying, and rocking can all help a woman during the process. Effleurage, the light massaging of the abdomen or thighs, is often soothing for the mothers.

63. Which of the following skin changes should the nurse highlight for a pregnant woman's health care practitioner? 1. Linea nigra. 2. Melasma. 3. Petechiae. 4. Spider nevi

3 There are many skin changes that occur normally during pregnancy. Most of the changes—such as linea nigra, melasma, and hyperpigmentation of the areolae—are related to an increase in the melanin-producing bodies of the skin as a result of stimulation by the female hormones estrogen and progesterone. The presence of petechiae is usually related to a pathological condition, such as thrombocytopenia.

25. The day after delivery, a woman, whose fundus is firm at 1 cm below the umbilicus and who has moderate lochia, tells the nurse that something must be wrong: "All I do is go to the bathroom." Which of the following is an appropriate nursing response? 1. Catheterize the client per doctor's orders. 2. Measure the client's next voiding. 3. Inform the client that polyuria is normal. 4. Check the specific gravity of the next voiding

3 This client's physical assessment is normal. If the client's bladder were distended, the client's fundus would be elevated in the abdomen and the client would have excess blood loss. It is unnecessary, therefore, either to catheterize the woman or to measure her output. Polyuria is normal because the client no longer needs the large blood volume she produced during her pregnancy.

14. There are four clients in active labor in the labor suite. Which of the women should the nurse monitor carefully for the potential of uterine rupture? 1. Age 15, G3 P0020, in active labor. 2. Age 22, G1 P0000, eclampsia. 3. Age 25, G4 P3003, last delivery by cesarean section. 4. Age 32, G2 P0100, first baby died during labor.

3 When babies are birthed via cesarean section, the surgeon must create an incision through the uterine body. The muscles of the uterus have, therefore, been ligated and a scar has formed at the incision site. Scars are not elastic and do not contract and relax the way muscle tissue does. A vaginal birth after cesarean (VBAC) section can be performed only if the woman had a low flap (Pfannenstiel) incision in the uterus during her previous cesarean section.

24. Infertility increases a client's risk of which of the following diseases? 1. Diabetes mellitus. 2. Nystagmus. 3. Cholecystitis. 4. Ovarian cancer

4 For a number of years, an association was noted between the long-term use of Clomid (clomiphene) to treat infertility and the incidence of ovarian cancer. It has also been shown that infertility itself is a contributing factor for ovarian cancer. The reason for the association is not yet known.

11. A primigravid client received Cervidil (dinoprostone) for induction 8 hours ago. The Bishop score is now 10. Which of the following actions by the nurse is appropriate? 1. Perform nitrazine analysis of amniotic fluid. 2. Report abnormal findings to the obstetrician. 3. Place woman on her side. 4. Monitor for onset of labor.

4 The Bishop score indicates the inducibility of the cervix of a client. Five signs are assessed—cervical position, cervical dilation, cervical effacement, cervical station, and cervical consistency. A total score is calculated. A primigravid cervix is considered inducible when the Bishop score is 9 or higher. A multigravid cervix is considered inducible when the Bishop score is 5 or higher.

87. A woman is pregnant. During amniocentesis it is discovered that her child has Down syndrome with a mosaic chromosomal configuration. She asks the nurse what that means. What is the nurse's best response? 1. "Instead of two number 21 chromosomes, your child has three." 2. "Your baby's number 21 chromosomes have black and white bands on them." 3. "Some of your baby's number 21 chromosomes are longer than others." 4. "Some of your baby's cells have two number 21 chromosomes and some have three."

4 The concept of mosaicism can be remembered by thinking about a mosaic piece of art. Mosaic tiles are bits of glass or ceramic that are different colors and shapes, but when put into a design create a piece of art. In genetics, mosaicism refers to the fact that various cells of the body have different numbers of chromosomes.

113. A macrosomic baby in the nursery is suspected of having a fractured clavicle from a traumatic delivery. Which of the following signs/symptoms would the nurse expect to see? Select all that apply. 1. Pain with movement. 2. Hard lump at the fracture site. 3. Malpositioning of the arm. 4. Asymmetrical Moro reflex. 5. Marked localized ecchymosis.

1 2 3 4 Clavicle breaks are a fairly common injury seen after a delivery. They usually result from a disproportion between the sizes of the maternal pelvis and the fetal body. Because shoulder dystocia is an obstetric emergency, threatening the life of the baby, obstetricians may purposefully break a baby's clavicle to enable the baby to be birthed as rapidly as possible.

69. A neonate is being assessed for necrotizing enterocolitis (NEC). Which of the following actions by the nurse is appropriate? Select all that apply. 1. Perform hemoccult test on stools. 2. Monitor for an increase in abdominal girth. 3. Measure gastric contents before each feed. 4. Assess bowel sounds before each feed. 5. Assess for anal fissures daily.

1 2 3 4 NEC is an acute inflammatory disorder seen in preterm babies. It appears to be related to the shunting of blood from the gastrointestinal tract, which is not a vital organ system, to the vital organs. The baby's bowel necroses with the shunting and the baby's once normal flora become pathological. Resection of the bowel is often necessary.

79. Which of the following questions should be asked of women during all routine medical examinations? Select all that apply. 1. "Has anyone ever forced you to have sex?" 2. "Are you sexually active?" 3. "Are you ever afraid to go home?" 4. "Does anyone you know ever hit you?" 5. "Have you ever breastfed a child?"

1 2 3 4 Women (or men) who are being abused rarely discuss their relationships unless asked directly. To identify clients who are being threatened, physically abused, and/or sexually abused, it is essential that nurses query them at each and every visit. The questioning can be done during a face-to-face interview or via a paper and pencil questionnaire. If the client states that he or she is being abused, the nurse should be ready to provide information on safe environments, police contacts, and the like. To be able to provide comprehensive care, the nurse must also know if his or her client is sexually active.

2. The physician has ordered oxytocin (Pitocin) for induction for 4 gravidas. In which of the following situations should the nurse refuse to comply with the order? 1. Primigravida with a transverse lie. 2. Multigravida with cerebral palsy. 3. Primigravida who is 14 years old. 4. Multigravida who has type 1 diabetes.

1 baby in the transverse lie is in a scapular presentation. The baby is incapable of being birthed vaginally. Whenever a vaginal birth is contraindicated, induction is also contraindicated.

54. It is time for a baby who is in the drowsy behavioral state to breastfeed. Which of the following techniques could the mother use to arouse the baby? Select all that apply. 1. Swaddle or tightly bundle the baby. 2. Hand express milk onto the baby's lips. 3. Talk with the baby while making eye contact. 4. Remove the baby's shirt and change the diaper. 5. Play pat-a-cake with the baby

2 3 4 5 It is important to distinguish a drowsy baby from a baby in the quiet alert or active alert state. For example, a baby who is in the active alert state may actually benefit from being swaddled because he or she is upset and needs to be calmed. Conversely, a baby in a drowsy state may need to be stimulated by manipulating or playing with the baby or by expressing milk onto the baby's lips

113. Which of the following behaviors should nurses know are characteristic of infant abductors? Select all that apply. 1. Act on the spur of the moment. 2. Create a diversion on the unit. 3. Ask questions about the routine of the unit. 4. Choose rooms near stairwells. 5. Wear over-sized clothing

2 3 4 5 The test taker should familiarize himself or herself with the many characteristics of the neonatal abductor including, in addition to those cited above, individuals who are emotionally immature, suffer from low selfesteem, and have a history of manipulative behavior.

42. A female client seeks care at an infertility clinic. Which of the following tests may the client undergo to determine what, if any, infertility problem she may have? Select all that apply. 1. Chorionic villus sampling. 2. Endometrial biopsy. 3. Hysterosalpingogram. 4. Serum progesterone assay. 5. Postcoital test.

2 3 4 5 There are a number of tests that are performed to assess fertility in couples. It is important to remember that many of the assessments are invasive, painful, embarrassing, and, depending on the results, may label one of the partners as the cause of the infertility. The knowledge of who is responsible for the infertility can be very difficult for some clients to learn.

44. A nurse is advising a mother of a neonate being discharged from the hospital regarding car seat safety. Which of the following should be included in the teaching plan? Select all that apply. 1. Place the baby's car seat in the front passenger seat of the car. 2. Position the car seat rear facing until the baby reaches two years of age. 3. Attach the car seat to the car at 2 latch points at the base of the car seat. 4. Check that the installed car seat moves no more than 1 inch side to side or front to back. 5. Make sure that there is at least a 3-inch space between the straps of the seat and the baby's body.

2 3 4 Test takers should be aware that recommendations and guidelines often change over time. In March 2011, the American Academy of Pediatrics came out with updated recommendations on infant and child seat restraint systems

87. At 28 weeks' gestation, an Rh-negative woman receives RhoGAM. Which of the following would indicate that the medication is effective? 1. The baby's Rh status changes to Rh-negative. 2. The mother produces no Rh antibodies. 3. The baby produces no Rh antibodies. 4. The mother's Rh status changes to Rh-positive

2 The test taker should review the immune response to an antigen. In this situation, the antigen is the baby's Rh+ blood. It can leak into the maternal bloodstream from the fetal bloodstream at various times during the pregnancy. Most commonly it happens at the time of placental delivery. Because the mother is antigen negative—that is, Rh-, when exposed to Rh+ blood, her immune system develops antibodies. RhoGAM is composed of Rh+ antibodies. It acts as passive immunity. Because antibodies are already present in the mother's bloodstream, her immune system is suppressed and fails to develop antibodies via the active immune response.

53. The nurse wishes to assess the variability of the fetal heart rate. Which of the following actions is recommended prior to performing this assessment? 1. Place the client in the lateral recumbent position. 2. Insert an internal fetal monitor electrode. 3. Administer oxygen to the mother via face mask. 4. Ask the mother to indicate when she feels fetal movement

2 There are many important principles related to electronic fetal heart monitoring. Variability is the most important of the baseline data. Variability is a measure of the competition between the sympathetic nervous system, which speeds up the heart rate, and the parasympathetic nervous system, which slows down the heart rate. When the fetal heart variability is adequate, the nurse can conclude, therefore, that the baby's autonomic nervous system is healthy

5. A woman's temperature has just risen 0.4°F and will remain elevated during the remainder of her cycle. She expects to menstruate in about 2 weeks. Which of the following hormones is responsible for the change? 1. Estrogen. 2. Progesterone. 3. Luteinizing hormone (LH). 4. Follicle-stimulating hormone (FSH)

2 This question asks the test taker to explain a temperature rise that occurs 2 weeks prior to menstruation. It is essential for the test taker to know that ovulation usually takes place approximately 14 days before the onset of the menses. That eliminates estrogen and FSH since the temperature would have been elevated much earlier in the cycle if those hormones were responsible. LH spikes and then drops at the time of ovulation; that hormone, too, can be eliminated as a possible answer. The only hormone that becomes elevated during the latter part of the menstrual cycle is progesterone.

9. The nurse turns off the oxytocin (Pitocin) infusion after a period of hyperstimulation. Which of the following outcomes indicates that the nurse's action was effective? 1. Intensity moderate. 2. Frequency every 3 minutes. 3. Duration 130 seconds. 4. Attitude flexed.

2 This question is asking the test taker to evaluate an expected outcome. When a nurse intervenes, he or she is expecting a positive outcome. In this situation, the nurse is determining whether or not the action has reversed the hyperstimulation that developed from oxytocin administration. The normal contraction frequency is evidence of a positive outcome.

25. Immediately prior to an amniotomy, the external fetal heart monitor tracing shows 145 bpm with early decelerations. Immediately following the procedure, an internal tracing shows a fetal heart rate of 120 with variable decelerations. A moderate amount of clear, amniotic fluid is seen on the bed linens. The nurse concludes that which of the following has occurred? 1. Placental abruption. 2. Eclampsia. 3. Prolapsed cord. 4. Succenturiate placenta.

3 The test taker must remember that variable decelerations are caused by cord compression. The fact that variables are seen in the scenario as well as a precipitous drop in the fetal heart baseline is an indirect indication that the cord is being compressed, resulting in decreased oxygenation to the fetus.

59. A baby has just been admitted into the neonatal intensive care unit with a diagnosis of intrauterine growth restriction (IUGR). Which of the following maternal factors would predispose the baby to this diagnosis? Select all that apply. 1. Hyperopia. 2. Gestational diabetes. 3. Substance abuse. 4. Chronic hypertension. 5. Advanced maternal age.

3 4 5 The test taker should be reminded that any condition that inhibits the flow of blood, including illicit drug use, hypertension, cigarette smoking, and the like, can lead to fetal IUGR—that is, a fetus smaller than expected for the gestational period.

77. A woman who is a carrier for sickle cell anemia is advised that if her baby has two recessive genes, the penetrance of the disease is 100%, but the expressivity is variable. Which of the following explanations will clarify this communication for the mother? All babies with 2 recessive sickle cell genes will: 1. Develop painful vaso-occlusive crises during their first year of life. 2. Exhibit at least some signs of the disease while in the neonatal nursery. 3. Show some symptoms of the disease but the severity of the symptoms will be individual. 4. Be diagnosed with sickle cell trait but will be healthy and disease-free throughout their lives.

3 The test taker must be familiar with common terms used to describe genetic diseases, like penetrance and expressivity. Penetrance: When a disease is 100% penetrant, 100% of the individuals who have the gene(s) for the disease will exhibit the disease. Similarly, if a disease is 80% penetrant, only 80% of the individuals who have the gene(s) for the disease will exhibit the disease. Expressivity: This term refers to the range of severity—or phenotypes—of a particular genetic disease.

84. A woman is to receive RhoGAM at 28 weeks' gestation. Which of the following actions must the nurse perform before giving the injection? 1. Validate that the baby is Rh-negative. 2. Assess that the direct Coombs' test is positive. 3. Verify the identity of the woman. 4. Reconstitute the globulin with sterile water.

3 When RhoGAM is given, the nurse is administering Rh antibodies to Rh- mothers. If the nurse should make a mistake and administer the dosage to an Rh+ mother, the client would then have been injected with antibodies that would act to destroy her own blood.

9. A client informs the nurse that she intends to bottle feed her baby. Which of the following actions should the nurse encourage the client to perform? Select all that apply. 1. Increase her fluid intake for a few days. 2. Massage her breasts every 4 hours. 3. Apply heat packs to her axillae. 4. Wear a supportive bra 24 hours a day. 5. Stand with her back toward the shower water.

4 5 The postpartum body naturally prepares to breastfeed a baby. To suppress the milk production, the mother should refrain from stimulating her breasts. Both massage and heat stimulate the breasts to produce milk. Mothers, therefore, should be encouraged to refrain from touching their breasts and when showering to direct the warm water toward their backs rather than toward their breasts. A supportive bra will help to minimize any engorgement that the client may experience.

14. Using the Neonatal Infant Pain Scale (NIPS), a nurse is assessing the pain response of a newborn who has just had a circumcision. The nurse is assessing a change in which of the following signs/symptoms? Select all that apply. 1. Heart rate. 2. Blood pressure. 3. Temperature. 4. Facial expression. 5. Breathing pattern

4 5 The student should be familiar with the pain-rating scales and use them clinically because neonates cannot communicate their pain to the nurse. The scoring variables that are evaluated when assessing neonatal pain using the NIPS are facial expression, crying, breathing patterns, movement of arms and legs, and state of arousal. Other pain assessment tools are the Pain Assessment Tool (PAT), the Neonatal Post-op Pain Scale (CRIES), and the Premature Infant Pain Profile (PIPP).

26. A multigravid client is 22 weeks pregnant. Which of the following symptoms would the nurse expect the client to exhibit? 1. Nausea. 2. Dyspnea. 3. Urinary frequency. 4. Leg cramping

4 : Although clients in the second trimester do experience some physical discomfort, such as leg cramps and backaches, most women feel well. They no longer are fatigued, nauseous, and so on as in the first trimester, but the baby is not so large as to cause significant complaints like dyspnea or the recurrence of urinary frequency

16. A woman, seen in the emergency department, is diagnosed with pelvic inflammatory disease (PID). Before discharge, the nurse should provide the woman with health teaching regarding which of the following? 1. Endometriosis. 2. Menopause. 3. Ovarian hyperstimulation. 4. Sexually transmitted infections.

4 The most common organisms to cause PID are the organisms that cause gonorrhea and chlamydia. In the early stages of these infections, women often experience only minor symptoms. It is not uncommon, therefore, for the organisms to proliferate and ascend into the uterus and fallopian tubes. The woman must be taught health care practices to decrease her likelihood of a recurrence of the problem

12. A 26-week-gestation woman is diagnosed with severe preeclampsia with HELLP syndrome. The nurse will assess for which of the following signs/symptoms? 1. Low serum creatinine. 2. High serum protein. 3. Bloody stools. 4. Epigastric pain.

4 When the liver is deprived of sufficient blood supply, as can occur with severe preeclampsia, the organ becomes ischemic. The client experiences pain at the site of the liver as a result of the hypoxia in the liver

19. The nurse is educating a group of adolescent women regarding sexually transmitted infections (STIs). The nurse knows that learning was achieved when a group member states that the most common sign/symptom of sexually transmitted infections is which of the following? 1. Menstrual cramping. 2. Heavy menstrual periods. 3. Flu-like symptoms. 4. Lack of signs or symptoms.

4 Women are usually symptom free when they initially contract gonorrhea or chlamydia. In addition, since the primary infection of syphilis, the chancre, is pain free, women may not realize they have been infected with the spirochete. As a result, it is very important that women, especially those with multiple sex partners, be seen yearly by a gynecologist or nurse practitioner to be tested for STIs.


Related study sets

Final Exam: GI Dysfunction NCLEX Questions

View Set

Chapter 9: Teaching Diverse Learners

View Set

Clinical Rotation & Didactic Year Exams

View Set

Ch. 3 Assessing the Internal Environment of the Firm

View Set

Principals and Practice - Math and Chapter 9

View Set

Biology 101G Ch.1 The Process of Science

View Set